Accueil Nutrition Instructions méthodologiques pour résoudre les problèmes et. Exemples et tâches au cours de la théorie de la combustion et de l'explosion Combustion 1 8

Instructions méthodologiques pour résoudre les problèmes et. Exemples et tâches au cours de la théorie de la combustion et de l'explosion Combustion 1 8

transcription

1 MINISTÈRE DE LA FÉDÉRATION DE RUSSIE POUR LA DÉFENSE CIVILE, LES URGENCES ET LE SECOURS DES CONSÉQUENCES DES CATASTROPHES NATURELLES Académie des pompiers d'État A.S. Androsov, E.P. Saleev EXEMPLES ET TÂCHES sur le cours THÉORIE DE LA COMBUSTION ET DE L'EXPLOSION Moscou 5

2 MINISTÈRE DE LA FÉDÉRATION DE RUSSIE POUR LA DÉFENSE CIVILE, LES URGENCES ET LE SECOURS DES CONSÉQUENCES DES CATASTROPHES NATURELLES Académie des pompiers d'État A.S. Androsov, E.P. Saleev EXEMPLES ET TÂCHES du cours THÉORIE DE LA COMBUSTION ET DE L'EXPLOSION Tutoriel Approuvé par le ministère de la Fédération de Russie pour la défense civile, les urgences et les secours en cas de catastrophe comme support pédagogique pour les établissements d'enseignement supérieur du ministère des Situations d'urgence de Russie Moscou 5

3 UDC BBK A ISB N Androsov A.S., Saleev E.P. Exemples et tâches pour le cours. Théorie de la combustion et de l'explosion. Didacticiel. - M. : Arfltvbz GPS EMERCOM de Russie, p. Évaluateurs : Département de chimie générale et spéciale de l'Académie des services d'incendie d'État du Ministère des situations d'urgence de Russie, Département du génie et des services d'incendie de l'Académie des services d'incendie d'État du Ministère des situations d'urgence de Russie. Des exemples et des tâches pour le cours Théorie de la combustion et de l'explosion sont compilés sur la base de nombreuses années d'expérience dans l'enseignement de la discipline à l'Académie des services d'incendie d'État du ministère des Situations d'urgence de Russie de manière à pouvoir servir de un guide pour la conception de cours. Afin d'assurer une unité méthodologique avec la partie théorique du cours, au début de chaque chapitre, des exemples de résolution de problèmes sont donnés, ainsi que des formules de calcul de base. L'annexe contient des tables de valeurs les plus couramment utilisées pour résoudre les problèmes de ce cours. Il est destiné aux cadets, étudiants et adjoints des établissements d'enseignement du ministère des Situations d'urgence de Russie d'un profil technique d'incendie. Les chapitres 1, 3 sont écrits par Ph.D. technologie. Sciences Professeur agrégé Androsov A.S., Chapitre 4 Cand. technologie. chercheur principal en sciences Saleev E.P. ISB N Academy of State Fire Service EMERCOM of Russia, 5

4 Sommaire Chapitre 1. Bilans matière et thermique des processus de combustion Calcul de la quantité d'air nécessaire à la combustion des substances Calcul du volume et de la composition des produits de combustion Calcul de la chaleur de combustion des substances Calcul des températures de combustion et d'explosion Chapitre. Limites de concentration de propagation de la flamme (allumage) Chapitre 3. Indicateurs de température du risque d'incendie Calcul des limites de température de propagation de la flamme (allumage) Calcul des températures d'éclair et d'inflammation Calcul de la température standard d'auto-inflammation. 61 Chapitre 4. Paramètres de l'explosion des mélanges vapeur-gaz Calcul de la pression maximale de l'explosion Calcul de l'équivalent TNT de l'explosion et de la distance de sécurité en fonction de l'action des ondes de choc dans l'air .. 63 Annexe

5 Chapitre 1. Bilans matière et thermique des processus de combustion La base théorique du calcul des bilans matière et thermique sont les lois fondamentales de la conservation de la matière et de l'énergie Calcul de la quantité d'air nécessaire à la combustion des substances Formules de calcul Pour les calculs pratiques, on suppose cet air est composé de 1% d'oxygène et d'azote. Ainsi, le rapport volumique d'azote et d'oxygène dans l'air sera : φ φ Ο 79 3,76, (1.1) 1 Ν où φ Ν, φ Ο, respectivement, la teneur volumique (% vol.) d'azote et d'oxygène dans le comburant environnement. Ainsi, pour 1 m 3 (kmol) d'oxygène dans l'air, il y a 3,76 m 3 (kmol) d'azote. Le rapport massique de l'azote et de l'oxygène dans l'air est de 3,3 % O et 76,7 % N. Il peut être déterminé à l'aide de l'expression : φ φ NOMMNO ,9, (1.) 1 3 où M,MON sont les masses moléculaires de l'oxygène et azote, respectivement. Pour faciliter les calculs, les substances combustibles sont divisées en trois types (tableau 1.1): composés chimiques individuels (méthane, acide acétique, etc.), substances de composition complexe (bois, tourbe, schiste, pétrole, etc.), un mélange de gaz (gaz de générateur, etc.). 5

6 Tableau 1.1 Type de substance combustible Formules de calcul Dimension Substance individuelle B (1.3, a) n non + n kmol m 3 ; N kmol m 3 Substance de composition complexe Mélange de gaz Г (no + nn) В ngm g (1.3, b) CSO В,69 + H (1.4) φg O i φo В in (1.5) 1 m 3 kg m 3 kg m 3 kmol; m 3 kmol Ici B est la quantité théorique d'air ; n G, non, nn la quantité de carburant, d'oxygène et d'azote obtenue à partir de l'équation de la réaction chimique de combustion, kmol; M G est le poids moléculaire du carburant ; le volume de 1 kmole de gaz dans des conditions normales (.4 m 3); Teneur massique en C, H, S, O des éléments correspondants dans la composition du carburant, % ; ϕ Г i concentration du i-ème composant combustible, % vol.; ϕ O concentration en oxygène dans le gaz combustible, % vol. ; non i la quantité d'oxygène nécessaire à l'oxydation d'un kmole du i-ème composant combustible, kmol. Pour déterminer le volume d'air pendant la combustion dans des conditions autres que la normale, utilisez l'équation d'état des gaz parfaits P T P T 1 1, (1.6) où P est la pression normale, Pa; T température normale, K; volume d'air dans des conditions normales ; P 1, 1, T 1 - respectivement pression, volume et température de l'air, caractérisant les conditions de combustion données. Quantité d'air pratique Le volume d'air qui entre réellement dans la zone de combustion. Le rapport du volume d'air pratique au volume théorique s'appelle le coefficient d'excès d'air α: 1 V α. (1.7) La différence entre les volumes d'air pratiques et théoriques est appelée excès d'air B : B B B B. (1.8) 6

7 Les équations (1.7) et (1.8) impliquent que В В (α-1). (1.9) Si la teneur en oxygène des produits de combustion est connue, le coefficient d'excès d'air est déterminé par la formule φ α 1+ O В (1 φ) O, (1.1) volume théorique des produits de combustion. Pour les substances dont le volume des produits de combustion est égal au volume d'air consommé (par exemple, le carbone), la formule (1.1) est simplifiée : 1 α. (1.11) 1 Dans le cas de la formation de produits de combustion incomplète (CO, H, CH 4, etc.), la formule (1.11) prend la forme φ O 1 α 1 φ, (1.11, a) + CH 4 où φ O, φ co, φ CH, φ 4 H teneur des substances correspondantes dans les produits de combustion, % vol. Si la teneur en oxygène dans le milieu oxydant diffère de sa teneur dans l'air, alors la formule (1.1) peut s'écrire : α 1+ et, par conséquent, la formule (1.11) φ O (φ φ) OOOO (1.1) φo α , (1.13) φ φ où φ O est la teneur initiale en oxygène dans le milieu oxydant, % vol.; volume théorique du milieu oxydant. Souvent, dans les calculs d'ingénierie incendie, il est nécessaire de déterminer la masse d'air qui est allée à la combustion, où ρ est la densité de l'air, kg / m 3. Évidemment, m in in ρ in, (1.14) 7

8 ρ φ М + φ М PT N N O O В.(1.15) PT 1 Après avoir substitué des valeurs constantes dans la formule (1.15), nous obtenons 3 P ρв 3, 47 1, (1.16) T où P est la pression atmosphérique, Pa; T température de l'air, K. Exemples Exemple 1. Déterminer la masse et le volume d'air théoriques nécessaires à la combustion de 1 m 3 de méthane dans des conditions normales. Solution Une substance combustible est un composé chimique individuel, par conséquent, la formule (1.3, a) doit être utilisée pour calculer le volume d'air. Nous écrivons l'équation de la réaction chimique de la combustion du CH 4 dans l'air CH 4 + O + 3,76 N CO + HO + 3,76 N. De l'équation, nous trouvons n O; n 3,76 7,5 ; N n 1 CH, puis 4 + 7,5 V 9,5 m 3 / m 3 ou kmol / kmol. 1 En utilisant la formule (1.14), en tenant compte de l'équation (1.15), nous calculons la masse d'air,79 8 +,1 3 m B 9,5 9,5 1,8 1, kg / m 3.,4 Déterminer le volume d'air théorique nécessaire à la combustion de 1 kg de benzène. Décision Un composé chimique individuel combustible, donc, à calculer selon la formule (1.3, b), nous écrivons l'équation de la réaction chimique de combustion C 6 H 6 + 7,5 O + 7,5 3,76 N 6 CO + 3 HO + 7,5 3,76 N, trouver n 1 ; n75 ; n 7,5 3,76 8,. O, N Poids moléculaire du benzène M Le volume de 1 kmole de gaz dans des conditions normales est de 4 m 3 (7,5 +,) 8,4 V 1,3 m 3 /kg

9 Exemple 3. Déterminer le volume et la masse d'air nécessaires à la combustion de 1 kg de masse organique de la composition : C 6 %, H 5 %, O 5 %, N 5 %, W 5 % (humidité), si l'excès coefficient air α.5; température de l'air 35 K, pression 995 Pa. Décision Étant donné que la substance combustible est de composition complexe, la quantité théorique d'air dans des conditions normales est déterminée par la formule (1.4) 6 5 V, 9 m 3 /kg. 3 8 À partir de la formule (1.7), nous calculons la quantité pratique d'air dans des conditions normales α, 5 5, m 3 /kg. En B, on trouve la quantité d'air qui est allée à la combustion d'une substance dans des conditions de combustion données. En utilisant la formule (1.6), on obtient m 14, V (RT) 16,8 m 3 / kg.8 1, (RT) V ρ V 18,9 V kg / kg. PRI me R 4. Déterminer le volume d'air requis pour la combustion de 5 m 3 de mélange de gaz, composé de% CH 4 ; 4%CH; 1 % de CO ; 5 % N et 5 % O si le taux d'air en excès est de 1,8. Solution Un mélange combustible de gaz, par conséquent, pour calculer le volume d'air qui est allé à la combustion, nous utilisons la formule (1.5). Pour déterminer les coefficients stoechiométriques de l'oxygène no i, on écrit l'équation des réactions de combustion des composants combustibles dans l'oxygène CH 4 + O CO + HO, CH +.5O CO + HO, CO +.5O CO, +, 5 4 +, puis B 5, 7m 3 / m 3. 1 Pour la combustion de 5 m 3 d'un mélange gazeux, le volume d'air théorique nécessaire sera de B 5 5, 7 8, 5 m 3. La quantité d'air pratique : 18 , 3, m 3. B, 9

10 Exemple 5. Déterminer le coefficient d'excès d'air lors de la combustion acide acétique si 3 m 3 d'air ont été fournis pour la combustion de 1 kg. Solution Pour déterminer le coefficient d'excès d'air à l'aide de la formule (1.7), il est nécessaire de calculer sa quantité théorique. Le poids moléculaire de l'acide acétique est de 6. CH 3 COOH + O + 3,76 N CO + HO + 3,76 N; (+,) 3 76,4 V 3,6 m 3 /kg. 1 6 Alors le coefficient d'excès d'air selon la formule (1.7) est égal à 3, α, 8. 3, 6 La combustion s'est déroulée avec un manque d'air. PRI mme R 6. Déterminer le volume d'air utilisé pour l'oxydation de 1 m 3 d'ammoniac, si la teneur en oxygène des produits de combustion était de 18 %. Décision Nous déterminons la quantité d'air théorique nécessaire à la combustion de 1 m 3 d'ammoniac : alors NH 3 +.75O +.75 3.76N.5N + 1.5HO +.75 3.76N, 75+, 75 3, 76 V 3, 6 m 3 /m 3. 1 Pour déterminer le coefficient d'excès d'air selon la formule (1.1), il est nécessaire de calculer la quantité théorique de produits de combustion 1 m 3 ammoniac (1., formule 1.14) 1,5 +,5 +, 75 3,76 4,8 m 3 /m 3. 1 Coefficient d'excès d'air 18 4,8 α 1+ 9,. 3, 6 1 (18) Le volume d'air impliqué dans le processus de combustion de 1m 3 d'ammoniac, nous le déterminons à partir de la formule (1.7) 9 3, m 3 /m 3. B, 1

11 Exemple 7. Déterminer le volume du milieu oxydant, composé de 6% O et 4% N, nécessaire à la combustion de 1 kg d'alcool isopropylique, si sa température est de 95 K, pression 6, kPa. Solution Étant donné que le milieu oxydant a une composition différente de celle de l'air, nous déterminons par la formule (1.1) le rapport volumique de l'oxygène et de l'azote à 4: 6,67. Équation de la réaction de combustion de l'alcool isopropylique C 3 H 7 OH + 4.5O + 4.5.67N 3CO + 4HO + 4.5.67N. Le volume théorique du milieu oxydant dans les conditions normales est calculé par la formule (1.3, b). Le poids moléculaire du carburant est de 6 : (4,5 + 4,5,67) 1 6,4 os.8 m 3 /kg. Le volume du milieu oxydant dans des conditions de combustion données est déterminé à partir de la formule (1.6) () 4.9 OS RT.35 6.73 m 3 /kg. Exemple 8. Déterminer la masse de dinitrotoluène, C 7 H 6 (NO), brûlé dans un volume étanche de 1 m 3, si la teneur en oxygène des produits de combustion était de 1 %. Solution Étant donné que les produits de combustion contiennent de l'oxygène, la combustion s'est déroulée dans un excès d'air. Le coefficient de dépassement est déterminé par la formule (1.1). C7H6(NO) + 6,5O + 6,5 3,76N7CO + 3HO + N + 6,5 3,76N. Masse moléculaire du combustible 18. Volume théorique d'air (6,5 + 6,5 3,76), 4 V 38, m 3 /kg Volume théorique des produits de combustion (formule 1.14) (, 5,) 3 76,4 4,4 m 3 /kg, 4 α 1 +, 1 (1) Volume pratique d'air utilisé pour la combustion, 55 38,9 7 m 3 /kg. À 11 HEURES

12 Puis la masse du dinitrotoluène brûlé m g est déterminée à partir du rapport P 1 m G 1,3 kg. 9.7 V Tâches de contrôle 1. Déterminez la masse et le volume (théoriques) d'air nécessaires à la combustion de 1 kg d'alcools méthylique, éthylique, propylique et amylique. Construire un graphique de la dépendance du volume d'air sur le poids moléculaire de l'alcool .. Déterminer le volume d'air théorique nécessaire à la combustion de 1 m 3 de méthane, éthane, propane, butane et pentane. Construisez un graphique de la dépendance du volume d'air à la position de la substance dans la série homologue (teneur en carbone dans la molécule de la substance). 3. Déterminez la masse d'air théorique utilisée pour la combustion de 1 kg de méthane, d'alcool méthylique, d'aldéhyde formique, d'acide formique. Expliquer la raison de l'influence de la composition d'une substance sur le volume d'air nécessaire à sa combustion. 4. Déterminer le volume et la masse d'air qui est entré en combustion de 1 kg de bois de la composition : C 47 %, H 8 %, O 4 %, W 5 %, si le coefficient d'excès d'air est de 8 ; pression 9 GPa, température 85 K. 5. Quelle quantité d'air, en kg, a été fournie à la combustion de 1 kg de carbone si la teneur en oxygène des produits de combustion était de 17 % ? 6. Quelle quantité d'air, en kg, doit être fournie pour la combustion m 3 de la composition du gaz du générateur : CO 9 %, H 14 %, CH 4 3 %, CO - 6,5 %, N - 45 %, O - 0,5 % , si le coefficient d'excès d'air est égal à 5 ? 7. Déterminez la quantité de toluène brûlé, en kg, dans une pièce d'un volume de 4 m 3 si, après un incendie en l'absence d'échange de gaz, il est établi que la teneur en oxygène a diminué à 17%. 8. Quelle quantité de chlore, m 3, a été fournie à la combustion de 3 m 3 d'hydrogène, si l'excès de comburant dans les produits de combustion était de 8 m 3 ? 9. Déterminer l'excès d'air dans les produits de combustion du mélange gazeux de composition : CO 15 %, C 4 H 1 45 % O 3 %, N 1 %, si le coefficient d'excès d'air est de 1,9. 1. Quelle quantité de milieu oxydant, m 3, composé de 5% d'oxygène et de 5% d'azote, est nécessaire pour la combustion de 8 kg d'acétate d'éthyle, si le coefficient d'excès est de 1 ; température 65 K, pression 85 GPa. 11. Déterminez le coefficient d'excès du milieu oxydant, composé de 7% d'oxygène et de 3% d'azote, si lors de la combustion du soufre, la teneur est de 1

13 oxygène est tombé à 55%. Déterminez la quantité de soufre brûlé (kg) si le volume de la pièce est de 18 m 3 Combien d'anthracite (en supposant que la teneur en carbone est de 1%) brûlée dans une pièce d'un volume de 15 m 3 si la combustion a cessé lorsque l'oxygène a diminué à 13 %. Les échanges gazeux sont ignorés. 13. Calculez le débit massique et volumique d'air nécessaire à la combustion d'une fontaine à gaz avec un débit de 3 millions de m 3 / jour, composé de CH 4 8%, CO 1%, HS 5%, O 5% à un température de l'air de 7 ° C et pression de 15 kPa. Devoir Calculer le volume et la masse du milieu oxydant nécessaire à la combustion de la ième substance combustible (tableau 1.). Numéro de variante Substance combustible Formule chimique Quantité de combustible Composition du milieu oxydant 1 Alcool méthylique CH 3 OH kg Air Aniline 3 Mélange de gaz 4 Nitrobenzène 5 Substance complexe C 6 H 7 N CO 45 % N 15 % C 4 H 8 1 % O 3 % 5 kg O 7% N 3% kg Air C 65 % O % H 5 % S 1 % T 8 KR 98 Pa α.5 g Air Normal α 1.4 6 Ethylène CH 4 5 m 3 O 5 % N 75 % 7 Soufre O 6 % S kg N 4 % 8 Substance complexe C 9 % H 3 % N 5 % O % 1 kg Air Normal α.5 T 35 K P1 Pa α 1.8 T 3 K P 95 Pa α 1.5 13

14 Numéro de variante 9 Mélange de gaz Substance combustible Formule chimique CH 4 15 % C 3 H 8 7 % O 1 % H 5 % comburant Conditions de combustion d'un milieu combustible 5 m 3 Air Normal α 1,9 1 Aluminium Al 15 kg O 4 % N 58% Normal α.8 11 Alliage Mg% Al 8% 8 kg Air T 65 KP 9 Pa α 1.5 1 Acide formique CHO 5 kg Air Normal α 1, 13 Diméthyléther (CH 3) O 1 kg Air 14 Mélange de gaz 15 Substance complexe 16 Glycérol HS 5 % SO 15 % CO 15 % H 3 % O 15 % C 8 % H 8 % W 1 % T 8 K P116 Pa α 4, 15 m 3 Air Normal α 1,4,7 kg Air C 3 H 8 O 3 1 kg Air 17 Acétylène CH 15 l Cl 18 % N 8 % 18 Mélange gazeux 19 Ester d'acide éthyl acétique Méthyléthyl cétone 1 Chlorobenzène Nitrotoluène CH 4 3 % O 8 % N 15 % H 47 % T 6 K P11 Pa α 1.4 T 35 K P113 Pa α 1.9 Normal α 1.8 3 m 3 Air Normal α 3, C 4 H 8 O 5 kg Air T 7 KR 85 Pa α 1.5 C 4 H 8 O 5 kg Air Normal α.5 T 35 KC 6 H 5 Cl 7 kg Air P 1 Pa α.8 C 7 H 7 NO 1 kg O 5% N 7 5% T 8 K R 98 Pa α 1,4 14

15 Variante numéro 3 Mélange de gaz Substance combustible Formule chimique NH 3 5 % C 4 H 1 5 % C 4 H 8 15 % CO 3 % O 5 % Quantité de combustible l Fin du tableau 1. Composition du milieu oxydant Conditions de combustion Air 4 Alcool butylique C 4 H 1 O 4 kg Air 5 Dibromohexane C 6 H 1 Br 3 kg 6 Substance composée 7 Mélange de gaz C 7 % S 5 % H 5 % O % C 3 H 8 1 % CO 79 % H 5 % O 5% N 1% O 65% N 35% 15 kg Air Normal α 1,8 T 65 K P1 Pa α 1,8 T 8 KP 98 Pa α 1,7 T 85 KR 1 Pa α.8 1 m 3 Air Normal α 3,5 1.. Calcul du volume et de la composition des produits de combustion Afin de simplifier le calcul, toutes les substances combustibles sont divisées en trois types : individuelles, complexes, mélanges de gaz combustibles (tableau 1.3) Type de substance combustible Substance individuelle Substance de composition complexe n Formules de calcul (1.17) ng ni (1.18) n M Ã C CO 1.86 1 (1.19) HWHO 11, + 1.4 1 1 (1.) S SO.7 1 ( 1.1) 1 7C+ 1 HO +,63S +,8 N (1 .) N 1 8 T ableau 1.3 Dimension m 3 ; kmol m 3 kmol m 3 kg m 3; kmol kg kg 15

16 Type de substance combustible Mélange de gaz i Formules de calcul i 1 ni φ n ii + φngi. 1 ng (1.3) Dimension m 3 ; kmol m 3 kmol Voici le volume théorique des produits de combustion ; n НГi la quantité du i-ème produit de combustion dans l'équation de réaction, kmol; n G quantité de carburant, kmol; volume de 1 kmole de gaz; M est le poids moléculaire du carburant ; NGi est le volume du ième produit de réaction ; Teneur en C, H, S, O, N, W des éléments correspondants (carbone, hydrogène, soufre, oxygène et azote) et humidité dans la substance combustible, % en poids ; ϕ Гi contenu du i-ème composant combustible dans le mélange gazeux, % vol. ; ϕ teneur en NGi du i-ème composant incombustible entrant dans la composition du mélange gazeux, % vol. Le volume pratique (total) des produits de combustion se compose du volume théorique des produits de combustion et de l'excès d'air ou + (α 1) + Δ (1,4) V B. (1,5) Composition des produits de combustion, c'est-à-dire la teneur en i-ème composant est déterminée par la formule φ i 1, (1.6) i i où ϕ i est la teneur en i-ème composant dans les produits de combustion, % vol. ; i volume du i-ème composant, m 3, kmol; Σ i volume total des produits de combustion, m 3, kmol. Lors de la combustion dans un excès d'air, les produits de combustion contiennent de l'oxygène et de l'azote O.1ΔВ; (1.7) N + 79, (1.8) N, B où N est le volume théorique d'azote dans les produits de combustion, m 3, kmol, 79. (1.9) N B 16

17 Exemples Exemple 1. Quelle quantité de produits de combustion sera libérée lors de la combustion de 1 m 3 d'acétylène dans l'air si la température de combustion était de 145 K. Solution Composé chimique individuel combustible (formule 1.17). Écrivons l'équation de la réaction chimique de combustion CH +.5O +.5 3.76N CO + HO +.5 3.76N Le volume des produits de combustion dans des conditions normales + 1+.5 3.76 1.4 m / m 3. 1 Le volume des produits de combustion à 145 K 1,4 145 (RT) 65,9 m3/m Ex. Déterminez le volume des produits de combustion lors de la combustion de 1 kg de phénol, si la température de combustion est de 1 K, la pression est de 95 Pa, le coefficient d'excès d'air est de 1,5. Solution Composé chimique individuel combustible (formule 1.18). Ecrivons l'équation de la réaction chimique de combustion C 6 H 5 OH + 7O + 7 3.76N 6CO + 3HO + 7 3.76N. Masse moléculaire du combustible 98. Volume théorique des produits de combustion dans des conditions normales () 3.76.4 8.1 m 3 /kg Volume d'air pratique dans des conditions normales (1.5) (.76) (1.5 1) 8.1 +, 4 11.9 m 3 /kg Le volume des produits de combustion dans des conditions spécifiées 11, (RT) 55,9 m 3 /kg Exemple 3. Déterminer le volume des produits de combustion lors de la combustion de 1 kg de masse organique de la composition : C 55 %, O 13 %, H 5 %, S 7 %, N 3 %, W 17 %, si la température de combustion est de 117 K, le coefficient d'excès d'air est de 1,3. 17

18 D olution Substance combustible de composition complexe (formules). Composition théorique des produits de combustion dans des conditions normales 55 CO 1,86 1. m 3 /kg ; H O 11, + 1.4.6 +.8 m 3 /kg; SO, 7,5 m 3 / kg; N,63+.83 4,7 m3/kg. 1 8 Volume total théorique des produits de combustion dans des conditions normales 1+.8 +.5 + 4.7 6.55 m 3 /kg. Le volume pratique des produits de combustion dans des conditions normales, 55+, (1,3 1) 6,55+ 1,8 8, 35 m 3 /kg. 3 8 Le volume pratique des produits de combustion à une température de combustion de 8, (RT) 35, 8 m 3 /kg. 73 Exemple 4. Calculer le volume des produits de combustion lors de la combustion de 1 m 3 d'un mélange gazeux composé de C 3 H 6 7%, C 3 H 8 1%, CO 5%, 15%, si la température de combustion est de 13 K, coefficient d'excès d'air, 8. Température ambiante 98 K. Solution Carburant - mélange de gaz (formule 1.3) C 3 H 6 + 4.5O + 4.5 3.76N 3CO + 3HO + 4.5 3.76N, C 3 H 8 + 5O + 5 3.76N 3CO + 4HO + 5 3.76N. Le volume des produits de combustion est déterminé par la formule (1.3) 1 CO (), 45 m 3 / m 3; 1 1 H O (), 4 m 3 / m 3. 1 Étant donné que le mélange gazeux contient de l'oxygène, il oxydera une partie des composants combustibles, par conséquent, la consommation d'air diminuera (formule 1.5). dix-huit

19 Dans ce cas, il est plus pratique de déterminer le volume théorique d'azote par la formule (1.9) 4, N,79 13, m 3 /m 3. 1 Volume théorique des produits de combustion, 45 +.4 + 13, 18.5 m 3 /m 3. Pratique le volume des produits de combustion (formules 1.4, 1.5) 4.5 + (.8 1) 4.5 m 3 / m 3. 1 Le volume des produits de combustion à une température de 13 K 4.5 13 (RT) 183.4 m 3 / m P Exemple 5. Déterminer la composition des produits de combustion de la méthyléthylcétone. Solution Avec une telle formulation du problème, il est plus rationnel de déterminer directement à partir de l'équation de combustion le volume de produits en kmol libérés lors de la combustion de 1 kmol de carburant CH 3 COC H 5 + 5,5O + 5,5 3,76N 4CO + 4HO + 5,5 3,76N, CO 4 kmol; 4 HO kmol; N, 7 kmoles; 7 et 8, priez. En utilisant la formule (1.6), on trouve la composition des produits de combustion 4 1 ϕ H 14 O ϕco %, ϕ N 1 () 7 %. 8.7 Exemple 6. Déterminer le volume et la composition (% vol.) des produits de combustion de 1 kg de composition d'huile minérale : C 85 %, H 15 %, si la température de combustion est de 145 K, le coefficient d'excès d'air est de 1,9. Décision À l'aide des formules (), nous déterminons le volume de produits de combustion 85 CO 1,86 1,6 m 3 /kg; 1 15 H O 11, 1,7 m 3 /kg; 1 1 N () 9,1 m 3 /kg. 1 Volume théorique des produits de combustion dans des conditions normales 19

20 1,6 + 1,7 + 9,1 1,4 m3/kg. Le volume pratique des produits de combustion dans des conditions normales (formule 1.5) 85 1.4 +, (1.9 1) 1.4 + 1.5, 9 m 3 /kg. 3 Le volume des produits de combustion à une température de 145 K.9 145 (RT) 11,7 m 3 /kg. 73 Évidemment, la composition des produits de combustion ne dépend pas de la température de combustion, il est donc raisonnable de la déterminer dans des conditions normales. Selon les formules (1.6, 1.8) 1.6 1.1 1.5 1 ϕ CO 7.1% ; ϕ 9,4 O %;.9,9 (9,1 + 0,79 1,5) 1 1,7 1 ϕ N 76, %; ϕ 7, 3 HO,9 %.,9 Exemple 7. Déterminer la quantité d'acétone brûlée, kg, si le volume de dioxyde de carbone libéré, ramené aux conditions normales, était de 5 m 3. Solution Écrivons l'équation pour le réaction de combustion de l'acétone dans l'air CH 3 COCH 3 + 4O + 4 3.76N 3CO + 3HO + 4 3.76N. Il résulte de l'équation que lors de la combustion, 3,4 m 3 de dioxyde de carbone sont libérés à partir de 58 kg (poids moléculaire de l'acétone). Alors, pour la formation de 5 m 3 de dioxyde de carbone, m G de combustible 5 58 m G 43, kg doivent réagir. 3.4 Exemple 8. Déterminer la quantité de masse organique brûlée de la composition : C 58 %, O %, H 8 %, N %, W 1 % dans une pièce d'un volume de 35 m 3, si la teneur en dioxyde de carbone était 5 %. Décision Déterminons le volume de dioxyde de carbone libéré Vyd 35,5 17,5 m3..1 m 3 / kg. un

21 Déterminons la quantité de substance brûlée 17,5 m Г 15,9 kg. 1.1 Exemple 9. Déterminez le moment où la teneur en dioxyde de carbone dans une pièce d'un volume de 48 m 3 à la suite de la combustion de bois (C 45%, H 5%, O 4%, W 8%) était de 8%, si le taux de combustion massique spécifique du bois est de 8 kg/(ms) et que la surface de combustion est de 38 m. Lors de la résolution, l'échange de gaz avec l'environnement n'est pas pris en compte, la dilution résultant de la libération de produits de combustion est négligée . Solution La dilution par les produits de combustion n'étant pas prise en compte, nous déterminons le volume de dioxyde de carbone dégagé à la suite de la combustion, correspondant à 8 % de sa teneur dans l'atmosphère 8 48 CO 38,4 m 3 1 à partir de l'expression (1.19) nous déterminer la quantité de matière combustible à brûler pour libérer un volume donné de dioxyde de carbone 38,4 m G 46 kg. 1,86,45 Le temps de combustion est déterminé sur la base du rapport m τ, υ Г F m où τ est le temps de combustion ; m G masse de bois brûlé, kg; υ m taux de combustion massique du bois, kg/(m s); surface de combustion F, m ; 46 τ 151 s.5 min.,8 38 Tâches de contrôle 1. Déterminer le volume et la composition (% vol.) des produits de combustion de 1 m 3 d'éthylène, de propylène, de butylène, si la température de combustion est de 18 K, la pression est de 98 Pennsylvanie. Construire un graphique de la dépendance du volume des produits de combustion et de la teneur en composants individuels sur le poids moléculaire du carburant .. Déterminer le volume des produits de combustion et la teneur en vapeur d'eau et en oxygène lors de la combustion de 1 kg d'hexane, d'heptane , octane, décane, si la température de combustion est de 13 K, la pression est de GPa, le coefficient d'excès est de 1

22 air comburant 1.8. Construire un graphique de la dépendance du volume des produits de combustion et de la teneur en oxygène sur le poids moléculaire du carburant. 3. Déterminez le volume et la composition des produits de combustion de 1 kg de bois avec la composition C 49%, H 6%, O 44%, N 1%, si la température de combustion est de 15 K, le coefficient d'excès d'air est de 1,6. 4. Combien de produits de combustion, réduits aux conditions normales, se forment à la suite de la combustion de 5 m 3 d'un mélange gazeux de composition H 45%, C 4 H 1%, CO 5%, NH 3 15%, o 15 %, si la combustion s'est déroulée à un coefficient d'excès d'air, égal à 3, ? 5. Déterminez la quantité de pétrole brut de la composition: C 85%, H 1%, S 5% brûlée dans un volume de 5 m 3, si la teneur en dioxyde de soufre était de 5 m 3. Calculez à quelle teneur en oxygène la combustion cessé. 6. Au bout de combien de temps la teneur en CO d'une pièce d'un volume de 3 m 3 résultant de la combustion d'hexanol d'une surface de 8 m sera-t-elle de 7 % ? Taux de combustion massique de l'hexane, 6 kg/(m s). 7. Déterminer la teneur en SO (% vol.) dans un volume de 1 m 3 par,5 m et 4 minutes d'huile brûlante de la composition : C 8 %, H 8 %, S 1 %, si son taux de combustion de une surface de 5 m était de 4 kg /(m s). Construire un graphique de la dépendance de la teneur en dioxyde de soufre sur le temps de combustion. 8. Déterminer le volume de produits de combustion libérés pendant 5 minutes après l'allumage du mélange gazeux de la composition : CH 3 %, H %, O 15 %, HS 18 %, CO 15 % et la teneur en dioxyde de carbone, si l'excès coefficient d'air est de 1,5, température de combustion 13 K. Consommation de gaz 5 m 3 / s, température des gaz 95 K. Devoirs Calculer le volume de produits formés, m 3, et leur teneur en azote (% vol.) lors de la combustion du i-ème (tableau 1.4). Tableau 1.4 Numéro de variante Substance combustible Formule chimique Quantité de combustible Composition du milieu oxydant 1 Alcool diéthylique (CH 5) O 1 kg Air Acide acétique CH 4 O 5 kg "3 Alliage Mg% Al 8% 1 kg" Conditions de combustion T g 15 K P114 Pa α.5 T g 1 KP 98 Pa α.6 T g 8 KP 95 Pa α 1.6

23 A suivre 1.4 Numéro d'option 4 Mélange de gaz Substance combustible Formule chimique CH 4 % C 3 H 8 65 % O 15 % Quantité de combustible Composition du milieu oxydant 1 m 3 "5 Alcool octylique C 8 H 18 O 1 kg" 6 Substance composée 7 Mélange de gaz 8 Aniline C 9% H 5% O 5% NH 3 1% C 4 H 1 8% N 7% O 3% 1 kg "1 m 3" C 6 H 7 N 1 kg "9 Éther diéthylique (CH 5) O 5 kg "1 Mélange de gaz 11 Nitrobenzène 1 Substance composée 13 Mélange de gaz CO 7 % C 3 H 8 5 % O 5 % C 6 H 5 NO C 7 % H 6 % O 14 % W 1 % CH 4 6% CO 3% H 1% 14 Diméthyléther (CH 3) O 1 kg 15 Glycérine 16 Substance complexe C 3 H 8 O 3 C 8% H 1% O 8% 1 m 3 O 4% N 58% kg Air 1 kg "1 m 3 "1 kg O 3% N 7% O 7% N 73% 1 kg Air Conditions de combustion T g 148 K P113 Pa α.4 T g 13 K P1 Pa α.5 T g 13 KP 97 Pa α 1,6 T g 16 K P113 Pa α 1, T g 155 K P 94 Pa α 1,7 T g 16 K P113 Pa α 1,4 T g 14 K P113 Pa α.5 T g 18 K P 87 Pa α 1,8 T g 13 KR 97 Pa α 1,3 T g 15 K R113 Pa α 1, T g 18 KR 87 Pa α 1,8 T g 16 K R113 Pa α.1 T g 135 KR 99 Pa α 1,8 3

24 À suivre 1.4 Numéro de variante 17 Mélange de gaz Substance combustible Formule chimique CH 6 6 % C 3 H 8 3 % H 5 % O 5 % Quantité de combustible Composition du milieu oxydant 1 m 3 - "- 18 Méthyléthylcétone C 4 H 8 O 1 kg - "- 19 Substance complexe Nitrotoluène 1 Mélange de gaz C 6% H 7% O - 1% W 1% 4 kg - "- C 7 H 7 NO kg - "- NH 3 4% C 3 H 8 4% H 1% O 1% Dibromohexane C 6 H 1 Br 1 kg 1 m 3 - "- O 5% N 5% 3 Dinitrobenzène C 6 H 4 (NO) 1 kg Air 4 Disulfure de carbone CS kg - "- 5 Dichlorobenzène C 6 H 4 Cl 5 kg - "- 6 Acide formique 7 Acétate d'éthyle C 7 % S 5 % H 5 % O % 1 kg O 8 % N % C 4 H 8 O 1 kg Air Conditions de combustion T g 165 K P113 Pa α. 6 T g 148 K R 91 Pa α 1,7 T g 11 K R113 Pa α 1,4 T g 134 K R1 Pa α,6 T g 18 K R113 Pa α 1,7 T g 14 K R 9 Pa α,3 T g 165 K R 81 Pa α 1,1 T 17 KR 97 Pa α 1,6 T 13 KR 99 Pa α 1,4 T 6 KR 98 Pa α,5 T g 15 K R1 Pa α 1,5 4

25 1.3. Calcul du pouvoir calorifique des substances Formules de calcul Lors du calcul du bilan thermique d'un incendie, en règle générale, le pouvoir calorifique inférieur est déterminé. La quantité de chaleur dégagée lors de la combustion d'une unité de masse (volume) de combustible à l'état gazeux de l'eau Q Q Q, B H où Q est le pouvoir calorifique le plus élevé ; Q n pouvoir calorifique inférieur ; Q est la chaleur d'évaporation de l'eau formée lors de la combustion d'une substance. c Type de combustible Substances Substances individuelles Substances de composition complexe (formule de Mendeleev) Mélange de gaz 9(OS) 5.1(9H + W) (1.31) 1 Q H QHφgi (1.3) 1 kJ/kg kJ/mol; kJ/m 3 où H i, H j, respectivement, la chaleur de formation d'un kmole du i-ème produit final de combustion et du j-ème matériau de départ ; n i, n j, respectivement, le nombre de kmoles du i-ème produit de réaction et du j-ème matériau de départ dans l'équation de réaction de combustion ; C, H, S, W, respectivement, contenu, % wt. carbone, hydrogène, soufre et humidité dans la composition de la substance; O somme d'oxygène et d'azote, % en poids ; Q H i pouvoir calorifique inférieur du i-ème composant combustible du mélange gazeux, kJ/kmol ; ϕ gi teneur du i-ème composant combustible dans le mélange gazeux, % vol. Le calcul du pouvoir calorifique des mélanges gaz-air s'effectue selon la formule CM 1 QHQ Hφ G, (1.33) 1 CM où QN est le pouvoir calorifique du mélange gaz-air, kJ / m 3, kJ / kmol ; Q N est le pouvoir calorifique inférieur d'une substance combustible, kJ / m 3, kJ / kmol; ϕ g concentration de carburant dans un mélange avec un comburant, % vol. 5

26 Le taux spécifique (intensité) de dégagement de chaleur pendant la combustion est q Q H υ M, (1.34) où q est l'intensité spécifique de dégagement de chaleur kW/m; υ m taux d'épuisement massique, kg/(m s). Le taux de dégagement de chaleur lors de la combustion, la chaleur du feu est égal à q Q H υ M F, (1,35) où q n est l'intensité du dégagement de chaleur, kW; Zone de combustion F, M. Exemples Exemple 1. Déterminez le pouvoir calorifique inférieur de l'acide acétique, si la chaleur de sa formation est de 485,6 kJ / mol. Solution Pour calculer à l'aide de la formule (1.3), nous écrivons l'équation de la combustion de l'acide acétique dans l'oxygène CH 3 COOH + O CO + HO; (396,9 + 4,1485,6) 79,6 3 Q kJ/mol 79,6 1 kJ/kmol. Н Pour calculer la quantité de chaleur dégagée lors de la combustion de 1 kg de carburant, il est nécessaire de diviser la valeur obtenue par son poids moléculaire (64) 3 79,6 1 Q Н 1384 kJ/kg. 64 EXEMPLE Calculer le pouvoir calorifique inférieur de la masse organique de la composition : C 6 %, H - 8 %, O 8 %, S %. D e x e n i e. Selon la formule de D.I. Mendeleïev (1,31) () 5, Q N 339,9 8 kJ/kg. Exemple 3. Déterminer le pouvoir calorifique inférieur d'un mélange gazeux composé de CH 4 4 %, C 4 H 1 %, O 15 %, HS 5 %, NH 3 1 %, CO 1 %. Solution Pour chaque composant combustible du mélange, en utilisant la formule (1.3), nous trouvons la chaleur de combustion (tableau 1.6). 6

27 Équation de chaleur pour la réaction de formation de carburant, 1-3 kJ/kmol CH 4 + O CO + HO 75 C 4 H 1 + 6.5O 4CO + 5HO 13.4 Q N Chaleur de combustion, 1-3 kJ/kmol T a b fa, 9 + 4, 75 86.3 Q 4 396, 13.5 H 666.1 HS + 1.5O HO + SO 1.1 Q 4, + 97.5 1.1 338.6 H NH 3 +.75O 1.5HO +.5N 46.1 Q 1.5 4, 46.1 317, N kJ/ kmol. Pour déterminer le pouvoir calorifique de 1 m 3 d'un mélange gazeux, il faut diviser la valeur obtenue par le volume occupé par 1 kmole de gaz dans des conditions standard (4,4 m 3): 3 178,5 1 QH 5776 kJ / m 3. 4,4 P p et mesurer 4. Calculer le pouvoir calorifique de 1 m 3 d'un mélange stoechiométrique hexane-air. Solution Nous trouvons la composition stoechiométrique du mélange combustible selon l'équation de la réaction de combustion C 6 H .5O + 9.5 3.76N 6CO + 7HO + 9.5 3.76N. Le volume total des composants ayant réagi (1 + 9,5 + 9,5 3,76) est pris comme 1 %, et la quantité de carburant (1 kmol) correspondra à la concentration stoechiométrique 1 1 φ G, %. 1+ 9,5 + 9,5 3,76 La chaleur de combustion de 1 m 3 d'hexane est déterminée par la formule (1,3) Q 6 396, 167, 399,6 kJ / mol, N 399,6 H 1 4,4 3 3 Q 1 16, kJ/m 3 . sept

28 Le volume d'un kmole de gaz dans des conditions standard est de 4,4 m 3. La chaleur de combustion de 1 m 3 d'un mélange stoechiométrique hexane-air est déterminée par la formule (1.33) 3 16, 1, Q 355 kJ / m 3 1 EXEMPLE 5. Déterminer l'intensité du dégagement de chaleur sur un feu de masse organique (composition dans l'exemple), si le taux de combustion est de 15 kg/(ms) et la surface du feu est de 15 m2. Solution. Selon la formule (1.35) : 3 q 646,5 kW 59,5 MW. P 1 Tâches de contrôle 1. Déterminez le pouvoir calorifique inférieur de 1 m 3 d'éthane, de propane, de butane, de pentane et d'hexane. Construire la dépendance de Q n sur le poids moléculaire du carburant. Chaleur de formation des substances combustibles : éthane 88,4 kJ/mol, propane 19,4 kJ/mol, butane 3,4 kJ/mol, pentane 184,4 kJ/mol, hexane 11, kJ/mol. Calculer la chaleur de combustion de 1 m 3 d'acétylène- mélange d'air aux limites de concentration inférieure et supérieure d'inflammation, ainsi qu'à la concentration stoechiométrique. Les limites d'inflammation en concentration (CPV) de l'acétylène sont égales à -81,%. N o t e : Tracez le pouvoir calorifique inférieur en fonction de la concentration de carburant dans l'air. Lors du calcul de la valeur calorifique du mélange au VKVV, il faut tenir compte du fait qu'une partie seulement du carburant peut être complètement oxydée dans l'air, le reste du carburant n'entrera pas dans une réaction de combustion en raison de l'absence d'un oxydant agent. 3. Déterminez le pouvoir calorifique inférieur de 1 kg de composition de bois C 49 %, H 8 %, O 43 %. Quelle est l'intensité spécifique du dégagement de chaleur dans un incendie si le taux de combustion massique est de 1 kg/(m·s) ? 4. Pour l'état du problème précédent, déterminez la variation du pouvoir calorifique et de l'intensité spécifique du dégagement de chaleur à une teneur en humidité du bois (supérieure à 1%) d'une valeur de 3, 5, 1 et 15%. Le taux de combustion du bois humide diminuera à 0,9, 8,6 et 0,5 kg/(m s) respectivement. Construire un graphique de Q n et q en fonction de la teneur en humidité d'un matériau combustible. Remarque Pour résoudre le problème, il est nécessaire de recalculer la composition du bois en tenant compte de l'humidité de manière à ce que la teneur de tous les composants soit de 1%. huit

29 5. Déterminer l'intensité du dégagement de chaleur, kW, lors de la combustion d'un mélange gazeux de composition : CO 15 %, C 4 H 8 4 %, O %, H 14 %, CO 11 %, si le taux d'expiration est de 8 m 3 / s Calcul de la température de combustion et d'explosion Température de température de combustion des produits de combustion dans la zone de réaction chimique. Il s'agit de la température maximale de la zone de flamme. Les températures de combustion et d'explosion sont déterminées à partir de l'équation du bilan thermique Q H n C i 1 p (v) i (TG T) Dans ce cas, la température de combustion adiabatique et la température de combustion réelle. (1.36) * QН TG T +, (1.37) C T Г T + pi Q C pi, (1.38) * où T Г et Т Г sont respectivement les températures de combustion adiabatique et réelle ; T - température initiale ; i volume du i-ème produit de combustion ; Q N est la plus faible chaleur de combustion d'une substance ; Q est la chaleur utilisée pour chauffer les produits de combustion ; C i est la capacité calorifique du i-ème produit de combustion à volume constant. Dans ce cas, Q Q Н (1 - η), (1.39) où η est la proportion de pertes de chaleur résultant du rayonnement énergétique, de la sous-combustion chimique et mécanique. Le calcul de la température de combustion à l'aide de la formule (1.37) ou (1.38) ne peut être effectué que par la méthode des approximations successives, car la capacité calorifique des gaz dépend de la température de combustion (tableau 1.7) 9

30 Paramètres déterminés p/n 1 Volume et composition des produits de combustion Pouvoir calorifique inférieur ou quantité de chaleur utilisée pour chauffer les produits de combustion (en présence de pertes de chaleur) 3 Valeur moyenne de l'enthalpie des produits de combustion 4 Température de combustion T 1 par enthalpie moyenne en utilisant le tableau 1a ou 1b , en se concentrant sur l'azote (teneur la plus élevée dans les produits de combustion) 5 Enchaleur des produits de combustion avec la température T 1 (tableau 1a, 1b annexe) 6 Si Q< Q Н (), то T >T 1 (in i (1.) kmol / kmol, m 3 / kg Q ou Q Ý (1,3) kJ / kmol, kJ / kg Remarque Tableau 1.7 QÝ () HCP (1,4) i 1 Q Hi i (1,41) H i - enthalpie du i-ème produit de combustion; i - / volume du i-ème produit de combustion si Q > QН (), alors T< T 1) 7 Q по формуле (1.41) 8 Расчет проводим до получения неравенства Q < QН () < Q 9 Температура горения (Н())(1) T Q Q T T T Г 1 + (1.4) Q Q Температура взрыва, протекающего в изохорно-адиабатическом режиме (при постоянном объеме) рассчитывается по уравнению теплового баланса (1.36) по методике, приведенной в табл Отличие заключается в том, что при расчетах вместо средней энтальпии продуктов горения и их теплосодержания (пп. 3-7) используется значение внутренней энергии газов (табл. приложения). Внутренняя энергия газов U C v T, где С v теплоемкость при постоянном объеме, кдж/(моль К), кдж/(м 3 К). Действительная температура горения на пожаре для большинства газообразных, жидких и твердых веществ изменяется в достаточно узких пределах (13-18 К). В связи с этим расчетная оценка действительной температуры горения может быть значительно упрощена, если теплоемкость продуктов горения выбирать при температуре 15 К: 3

31 Qн TG T +, (1.43) * С * où C Pi est la capacité calorifique du i-ème produit de combustion à 15 K (tableau 1.8). Substance kJ/(m 3 K) Pi i Capacité calorifique ,4 31, Air 1,44 3,6 1-3 Exemples Exemple 1. Déterminer la température adiabatique de la combustion de l'alcool éthylique dans l'air. Décision Le calcul est effectué selon le schéma indiqué dans le tableau Depuis la substance individuelle combustible, pour déterminer le volume et la composition des produits de combustion, nous écrivons l'équation de la réaction chimique de combustion CH 5 OH + 3O + 3 3.76N CO + 3HO + 3 3,76N. Par conséquent, les produits de combustion sont constitués de : CO mol, HO 3 mol, N 11,8 mol, 16,8 mol. D'après le tableau 3 de la demande, nous trouvons la chaleur de formation du carburant - 78, kJ / mol Q H 396, - 78, 14, kJ / mol. 3. Enthalpie moyenne des produits de combustion 14, H cf 76,3 kJ/mol. 16,8 4. Étant donné que H cf est exprimé en kJ / mol, selon le tableau. 1a de l'application, nous sélectionnons, en nous concentrant sur l'azote, la première température de combustion approximative T 1 1 o C. 5. Calculez le contenu calorifique des produits de combustion à 1 o C à l'aide de la formule (1.41) Q 114,7 + 93,4 11,8 133,7 kJ / mol. 31

32 6. Comparez Q H et Q, puisque Q > QH, choisissez la température de combustion égale à o C. 7. Calculez l'enthalpie des produits de combustion à o C : Q 18,6 + 88,1,8 11,8 135 kJ/mol. 8. Depuis Q< Q < Q, определим температуру горения по формуле (1.4) Н (14, 135)(1) T + 1 о С. 133,7 135 Г П р и м е р. Определить адиабатическую температуру горения органической массы, состоящей из С 6 %, Н 7 %, О 5 %, W 8 %. Р е ш е н и е. 1. Так как горючее представляет собой сложное вещество, состав продуктов горения рассчитываем по формулам () 6 CO 1,86 1,1 м /кг; 1,4, 88 H O 11, 1 + м 3 /кг; N ,1 м 3 /кг. 1 8 Общий объем продуктов горения равен 7, 1 м3 /кг.. Определим низшую теплоту cгорания вещества по формуле Д.И. Менделеева (1.31) Q Н 339,9 5-5,1() 3958,4 кдж/кг. 3. Определим среднюю энтальпию продуктов горения 3958,4 H CP 3417,7 кдж/м 3. 7,1 4. Так как величина энтальпии рассчитана в кдж/м 3, первую приближенную температуру выбираем по табл. 1б приложения. Ориентируясь на азот, принимаем Т 1 1 о С. 5. Рассчитываем теплосодержание продуктов горения при 1 о С по формуле (1.41) Q 5118, 1,1,9 5,1 5144,5 кдж/кг 6. Из сравнения Q Н и Q Q Н >Q choisir la deuxième température approximative égale à 19 o C. 7. Calculer l'enthalpie des produits de combustion à 19 C 3

33 Q 4579,7 1,5, 5,1 498,8 kJ/kg. 8. Depuis Q< QН < Q, определим температуру горения (3958,4 498,8)(1 19) T Г о С,8 П р и м е р 3. Рассчитать действительную температуру горения фенола (H обр 4, кдж/моль), если потери тепла излучением составили 5 % от Q н, а коэффициент избытка воздуха при горении,. Р е ш е н и е. 1. Определим состав продуктов горения: C 6 H 5 OH + 7O + 7 3,76N 6CO + 3H O + 7 3,76N, 6 моль; 3 моля; 6, 3 моля, CO H O (,76)(, 1) 39, 98 N В моля, 75, 3 моля.. Определим низшую теплоту сгорания фенола (формула 1.3): Q Н 7 396, - 1 4, 35,7 кдж/моль, так как по условию задачи 5 % тепла теряется, определим количество тепла, пошедшее на нагрев продуктов горения (теплосодержание продуктов горения при температуре горения) (формула 1.39) Q 35,7(1 -,5) 65,5 кдж/моль. По формуле (1.43) определим действительную температуру горения 65,5 Т Г К. 3 1 (5,81 6,3+ 3,6 39,98) П р и м е р 4. Рассчитать температуру взрыва метановоздушной смеси стехиометрического состава. Р е ш е н и е. Расчет проводим по схеме, представленной в табл Объем и состав продуктов горения СН 4 + О + 3,76N СО + Н О + 3,76N. Продукты горения: CO 1 кмоль/кмоль, H O моль/моль, N 3,76 7,5 кмоль/кмоль.. Низшая теплота сгорания: Q Н 1 396,6 + 4, кдж/моль. 3. Средняя внутренняя энергия продуктов горения QН 86 U ср 76,8 кдж/моль. 1,5 33

34 4. Selon le tableau. application, nous prenons la première température d'explosion approximative (pour l'azote) T 1 7 o C. 5. Calculez l'énergie interne des produits de combustion à T 1: U 1 pgi U i 1 18,9 + 1,4 + 7,5 7,86, kJ / mol. 6. La comparaison de la valeur de Q H et U 1 montre que T 1 est trop élevé. 7. Choisissez T 5 o C. U 1 118,3 + 94,3 + 7,5 64,3 789, kJ / mol. 8. Puisque U 1 > Q H > U T vzr, (7 5) 54 o C. 86, 789, Tâches de contrôle 1. Déterminer comment la température de combustion adiabatique change dans la série homologue d'hydrocarbures saturés (par exemple, méthane, propane, pentane et heptane). Construire un graphique de la dépendance de la température de combustion sur le poids moléculaire de la substance combustible Déterminer comment la température de combustion adiabatique de la composition du bois change : C 49 %, H 8 %, O 43 %, si la teneur en humidité (plus de 1 % ) est de 5, 15 %. Construire un graphique de la dépendance de la température de combustion sur la teneur en humidité du combustible. Remarque Lors de la résolution du problème, il est nécessaire de recalculer la composition du bois afin que la quantité de tous les composants (y compris l'eau) soit de 1%. 3. Déterminez comment la température adiabatique de la combustion du benzène changera dans l'air et dans un environnement oxydant contenant 5, 3 et 4 % d'oxygène. Construire un graphique de la dépendance de la température de combustion sur la teneur en oxygène. 4. Calculez la température de combustion réelle d'un mélange gazeux composé de 45% H, 3% C 3 H 8, 15% O, 1% N, si la perte de chaleur était de 3% de QH et que le coefficient d'excès d'air pendant la combustion est 1.8 . 5. Déterminez la quantité d'anthracite brûlé (C 1%) dans une pièce d'un volume de 18 m 3, si la température volumique moyenne est passée de 35 à 65 K. 98,1% d'air) si la perte de chaleur par rayonnement est % de la valeur calorifique nette. 34

35 7. Déterminez comment la température de combustion de l'acétylène changera lorsqu'il est dilué avec de l'azote à raison de 1,3%, si la perte de chaleur par rayonnement est de 5% du pouvoir calorifique inférieur, le coefficient d'excès d'air est de 1. Tracer la température en fonction de la teneur en azote dans l'acétylène. 8. Déterminez le temps de combustion du toluène, auquel la température dans une pièce d'un volume de 4 m 3 passera de 95 à 375 K, si son taux de combustion est de 15 kg / (ms) et la zone d'incendie est de 5 m Lors du calcul, négligez l'augmentation du volume des produits de combustion par rapport à l'air consommé. Devoir Calculez la température de combustion de la ième substance (tableau 1.9). Numéro d'option Substance combustible Formule chimique Composition du milieu oxydant 1 Mélange de gaz CO 4 %, C 3 H 8 5 %, CO 1 % Air T a b l e 1.9 Conditions de combustion α 1,4 η.5 Substance C 8 %, H 5 %, α 1.6 composition complexe S 6%, W 9% - "- η.3 3 Acide propionique C 3 H 6 OO 5%, N 75% α 1.3 η.4 4 Glycérine C 3 H 8 O 3 Air α 1, η,35 5 Éther butylique acétique C 6 H 1 O - "- α 1,4 η,15 6 Éthylbenzène C 8 H 1 - " - α 1,5 η, 7 Substance de composition complexe C 8%, H 8%, O 5%, W 5% - "- α 1, η,35 8 Mélange gazeux CO 6 %, H 4 % - " - α 1,8 η.4 9 Ammoniac NH 3 - " - α 1, η , 1 Hexane C 6 H 14 - " - α 1,4 η,15 11 Nitroéthane CH 5 NO - "- α 1,5 η, 1 Alcool hexylique C 6 H 14 O Air α, η,1 35

36 Numéro de variante Fin du tableau 1.9 Substance combustible Formule chimique Composition du milieu oxydant Conditions de combustion C 75 %, H 8 %, - « - α 1, composition complexe C 1 %, W 5 % η,4 14 Substance 15 Mélange de gaz CH 4 7 %, NH 3 %, O 1 % 16 Acide formique 17 Substance de composition complexe 18 Substance de composition complexe CH OO 5 %, N 75 % C 56 %, H 14 %, O %, W 1 % C 78 %, H 1 %, O 1% 19 Mélange de gaz CO 75%, CH 4 5% Mélange gazeux C 3 H 8 7%, C 4 H 1%, O 1% C 85%, H 1%, O 5% 1 Substance de composition complexe Mélange gazeux CH 6 75 %, CH 4 %, O 5 % 3 Substance de composition complexe C 7 %, H 16 %, O 14 % 4 Mélange de gaz CO 5 %, CH 4 3 %, CO % 5 Substance de composition complexe C 77 %, H 13 %, N 4 %, O 6 % - "- α 1,8 η, α, η.3 Air α 1, η.4 - "- α 1,6 η.15 - "- α 1,9 η, - "- α 1,8 η, - "- α 1,4 η,3 - "- α 1,7 η, - "- α 1, η,35 - "- α 1,9 η,15 - "- α 1, η,45 6 Éthylène CH 4 O 3% N 7% α 1,5 η.4 7 Alcool amylique C 5 H 1 O Air α, η.15 36

Chapitre 37 Limites de concentration de propagation de flamme (allumage) La limite inférieure (supérieure) de concentration de propagation de flamme est la concentration minimale (maximale) de combustible dans le comburant qui peut s'enflammer à partir d'une source à haute énergie avec une combustion ultérieure propagée à l'ensemble du mélange. Formules de calcul La limite inférieure de concentration d'inflammation ϕ H est déterminée par le pouvoir calorifique limite. Il a été établi que 1 m 3 de divers mélanges gaz-air au PCIV dégage lors de la combustion une quantité moyenne constante de chaleur de 183 kJ, appelée chaleur limite de combustion. Par conséquent, φ Q 1 PR N, (.1) QН si nous prenons la valeur moyenne de Q PR égale à 183 kJ / m 3, alors ϕ Н sera égal à φ Н Q où Q Н est le pouvoir calorifique inférieur du substance combustible, kJ / m 3. Les CPV inférieur et supérieur peuvent être déterminés par la formule d'approximation H ϕ () 1 H B, (.) an + b où n est le coefficient stoechiométrique à l'oxygène dans l'équation de la réaction chimique; a et b sont des constantes empiriques dont les valeurs sont données dans le tableau..1. T a b l e.1 Limites de concentration Valeurs d'inflammabilité a b Inférieur 8.684 4.679 Supérieur n 7.5 1.55.56 n > 7.5.768 6.554 37

38 Les limites de concentration d'inflammation des vapeurs de substances liquides et solides peuvent être calculées si les limites de température sont connues φ Н (В) pн(В) 1, (.3) p ) limite d'inflammation, Pa; p - pression ambiante, Pa. La pression de vapeur saturante peut être déterminée à partir de l'équation d'Antoine ou du tableau. 4 annexes B lg P A, (.4) С + t où A, B, C sont des constantes d'Antoine (tableau 1 de l'annexe) ; t - température, C (limites de température). Pour calculer les limites de concentration d'inflammation des mélanges de gaz combustibles, la règle de Le Chatelier est utilisée où φ P 1 n () CM 1 φ H (V), (.5) μ i φ N (V) i V inférieur (supérieur ) CPV d'un mélange de gaz,% environ.; ϕ н(в)i - Н(В) Р limite inférieure (supérieure) d'inflammation du i-ème gaz combustible %, vol.; µ i - fraction molaire du i-ème gaz combustible dans le mélange. Il faut garder à l'esprit que Σµ i 1, c'est-à-dire, la concentration des composants combustibles du mélange gazeux est prise égale à 1 %. Si les concentrations limites d'inflammation sont connues à une température T 1, alors à une température T elles sont calculées par les formules (.6), (.7) φ, limite inférieure d'inflammation en concentration, respectivement, aux températures Т et Т 1 ; φ VG et 1 φ VG limite supérieure de concentration d'inflammation, respectivement, aux températures T 1 et T ; T G température de combustion du mélange. 38

39 Approximativement, lors de la détermination de la LIE de TG, 155 K sont pris, lors de la détermination du VKVL, 11 K. Lorsque le mélange gaz-air est dilué avec des gaz inertes (vapeurs N, CO, HO, etc.), la région d'allumage se rétrécit : la limite supérieure diminue et la limite inférieure augmente. La concentration d'un gaz inerte (phlegmatisant), à laquelle les limites inférieure et supérieure d'inflammation sont fermées, est appelée concentration minimale flegmatisante ϕ f. La teneur en oxygène dans un tel système est appelée teneur minimale en oxygène explosif MVSK O) la teneur en oxygène en dessous de la MVSK est appelée sans danger des paramètres spécifiés est effectuée selon les formules О sans Calcul 1 ; (.8) 1 φf ; ( .9) 4.844 φ,φ 4, (.1) 1 O où ΔH f est la chaleur standard de formation du carburant, J/mol ; hi, h" i, h selon le type d'élément chimique dans la molécule de carburant et le type de flegmatisant (tableau 11 de l'annexe), mi est le nombre d'atomes du i-ème élément (groupe structurel) dans la molécule de carburant Le calcul de ces paramètres peut être effectué en utilisant une autre méthode physiquement plus transparente en résolvant le bilan thermique ( 1.36) sous les deux conditions suivantes : - au point de flegmatisation, le mélange combustible a une température limite de combustion de 15 K, - le mélange est stoechiométrique lorsque le carbone est oxydé en CO, l'hydrogène en HO. (1.36) dans le cas d'une dilution avec un gaz neutre est représenté par : Q Í (Т) Ã Т С ni + Срф nф Рi, (.11) * où Т Г est la température limite de combustion 15 K; C Pi, C Rf, respectivement, la capacité calorifique du i-ème produit de combustion et du gaz neutre (phlegmatisant), kJ / (mol K); n i le nombre de moles du ième produit de combustion du mélange stoechiométrique, mol/mol ; n f est le nombre de moles de gaz neutre au point du flegmatiseur, mol/mol. 39

40 De (.11) n Q (TGT) СРi С (Т Т) N f RF Г ni (.1) En prenant le volume de tous les composants du mélange gaz-air à 1 %, la concentration (% vol.) chacun d'eux ni φi 1 (.13) n + n + n + n à ON f Exemples Exemple 1. Déterminer la limite inférieure de concentration d'inflammation du butane dans l'air à partir de la chaleur limite de combustion. Décision Pour le calcul selon la formule (.1) du tableau. 3 applications on trouve le pouvoir calorifique le plus bas d'une substance 88,3 kJ/mol. Cette valeur doit être convertie en une autre dimension kJ/m 3 : 88, kJ/m 3., 4 En utilisant la formule (.1), nous déterminons le LEQV φ H 1,4 %. 18,7 13 Selon le tableau. 4 applications, nous constatons que la valeur expérimentale de ϕ Н est de 1,9 %. L'erreur relative de calcul s'élevait donc à 1,9 1,4 H 1 5 %. 1.9 EXEMPLE Déterminer les limites de concentration d'inflammation de l'éthylène dans l'air. Élution Le calcul du CPV est effectué selon la formule d'approximation. Nous déterminons la valeur du coefficient stoechiométrique pour l'oxygène Ainsi, n 3, puis C H 4 + 3O CO + H O. 1 φ H 3,5 8,679 % ; 1 φ 18, 1,55 3,56 3 V + %. Déterminons l'erreur relative de calcul. D'après le tableau 4 applications les valeurs limites expérimentales sont 3, 3, : 4


MINISTÈRE DE L'ÉDUCATION ET DES SCIENCES DE LA FÉDÉRATION DE RUSSIE Université technologique d'État de Belgorod. V.G. THÉORIE DE LA COMBUSTION ET DE L'EXPLOSION de Shukhova Directives pour la réalisation exercices pratiques

Thème 4 "VAPEURS ET MÉLANGES GAZ-AIR DANGEREUX D'INCENDIE ET ​​D'EXPLOSION" Leçon 4.2 "Calcul des limites d'inflammabilité en concentration" (2 heures) Formules de calcul Limite inférieure d'inflammabilité en concentration (LIE) n

CALCULS THERMIQUES DANS LES PROCÉDÉS DE COMBUSTION 1. PROPRIÉTÉS DES SUBSTANCES COMBUSTIBLES Selon la composition des phases, les substances combustibles peuvent être liquides, solides et gazeuses. Pouvoir calorifique (pouvoir calorifique) du carburant Q

Travail de contrôle sur la discipline "Théorie de la combustion et des explosions" Option 1 (kJ / kg) d'un composé individuel de toluène (C 6 H 5 CH 3). 2. Déterminer le volume d'air nécessaire à la combustion de 1 kg de benzène (l)

EMERCOM DE RUSSIE ÉTABLISSEMENT D'ENSEIGNEMENT SUPÉRIEUR PROFESSIONNEL UNIVERSITÉ DE SAINT-PÉTERSBOURG SERVICE D'INCENDIE D'ÉTAT EMERCOM DE RUSSIE CHIMIE DES PROCÉDÉS

MINISTÈRE DE L'ÉDUCATION ET DES SCIENCES DE LA FÉDÉRATION DE RUSSIE Établissement d'enseignement budgétaire d'État de l'enseignement professionnel supérieur "PENZA STATE UNIVERSITY" THÉORIE DE LA COMBUSTION ET DE L'EXPLOSION Stage

AGENCE FÉDÉRALE DES TRANSPORTS FERROVIAIRES Département des transports ferroviaires de l'Université d'État de l'Oural "Sécurité technosphérique" A. Zh. Khvorenkova THÉORIE DE LA COMBUSTION ET DE L'EXPLOSION Collection de tâches dans la direction

MINISTÈRE DE LA FÉDÉRATION DE RUSSIE POUR LA PROTECTION CIVILE, LES URGENCES ET LE SECOURS DES CONSÉQUENCES DES CATASTROPHES NATURELLES Académie des pompiers d'État I. R. Begishev THÉORIE DE LA COMBUSTION

MINISTÈRE DE L'ÉDUCATION ET DES SCIENCES DE LA RUSSIE Établissement d'enseignement budgétaire de l'État fédéral de l'enseignement professionnel supérieur "Université technique d'État d'Omsk"

MINISTÈRE DE LA FÉDÉRATION DE RUSSIE POUR LA PROTECTION CIVILE, LES URGENCES ET LE SECOURS DES CONSÉQUENCES DES CATASTROPHES NATURELLES Académie des pompiers de l'État I. R. Begishev COURS

Fours tubulaires à combustible. Calcul du processus de combustion du carburant Informations générales sur le carburant Le carburant est une matière organique qui est brûlée pour produire de la chaleur. Les principaux composants combustibles du carburant sont

Yu.S. Biryulin, V.N. Mikhalkin CALCUL THERMODYNAMIQUE DE LA CHALEUR DE COMBUSTION DES HYDROCARBURES La chaleur de combustion est importante pour évaluer le risque d'incendie des substances, et est également un indicateur de

Thème 2 "BILAN THERMIQUE ET MATÉRIEL DES PROCESSUS DE COMBUSTION ET D'EXPLOSION" Leçon 2.2 "Bilan thermique des procédés de combustion" 1 Questions à l'étude : 1. Chaleur de combustion. 2. Température de combustion. Littérature : 1.

O.V. Arkhangelskaya, I.A. Tyulkov M. V. Lomonossov Tâche difficile ? Commençons dans l'ordre Comme le montre la pratique, la thermochimie est l'une des sections les plus difficiles de la chimie pour les candidats. Pour la résolution de problèmes

17. Modèles de processus chimiques. Le concept de la vitesse d'une réaction chimique. Facteurs affectant la variation de la vitesse d'une réaction chimique La vitesse d'une réaction chimique est le rapport de la variation de concentration

Agence fédérale des transports ferroviaires Université d'État des transports ferroviaires de l'Oural Département de la sécurité des personnes N. V. Gushchina THÉORIE DE LA COMBUSTION ET DE L'EXPLOSION Iekaterinbourg 11 Fédéral

MINISTÈRE DE L'ÉDUCATION ET DES SCIENCES DE LA FÉDÉRATION DE RUSSIE Établissement d'enseignement budgétaire de l'État fédéral de l'enseignement professionnel supérieur "RECHERCHE NATIONALE TOMSK POLYTECHNICAL

AGENCE FÉDÉRALE POUR L'ÉDUCATION Établissement d'enseignement budgétaire de l'État de l'enseignement professionnel supérieur Module "RECHERCHE NATIONALE DE L'UNIVERSITÉ POLYTECHNIQUE DE TOMSK"

Pouvoir calorifique et température de combustion du combustible Leçon 3 Combustibles Le combustible est une source d'énergie ; une substance combustible qui produit une quantité importante de chaleur lors de la combustion Combustible solide : naturel

MINISTÈRE DE L'ÉDUCATION ET DES SCIENCES DE LA FÉDÉRATION DE RUSSIE Université technique d'État de Bryansk APPROUVÉ par le recteur de l'Université O.N. Fedonin 2014 FOURS D'ATELIERS DE FONDERIE CALCUL DES CARACTERISTIQUES DE COMBUSTION

EMERCOM DE RUSSIE UNIVERSITÉ DES POMPIERS D'ÉTAT DE SAINT-PÉTERSBOURG Korobeynikova E.G. THÉORIE DE LA COMBUSTION ET DE L'EXPLOSION

Thème 5 Détermination des conditions de sécurité pour l'utilisation des bouteilles de gaz inflammables Objectif : acquérir des compétences pratiques dans la réalisation de calculs techniques pour évaluer les conditions de sécurité pour l'utilisation des bouteilles

MINISTÈRE DE LA FÉDÉRATION DE RUSSIE POUR LA DÉFENSE CIVILE, LES URGENCES ET LE SECOURS DES CONSÉQUENCES DES CATASTROPHES NATURELLES Académie des pompiers de l'État THÉORIE DE LA COMBUSTION ET DE L'EXPLOSION

Option 1 1 2 3 4 Carbure de calcium acétylène benzène nitrobenzène aniline 5 6 éthylène éthanol 3. Quel volume d'air est consommé pour brûler 25 litres de méthylamine contenant 4 % d'impuretés incombustibles ? Fraction volumique d'oxygène

UDC 64.84.4 I.O. Stoyanovitch, V.S. Saushev, Le Xuan Ty (Russie, Vietnam) MÉTHODES DE CALCUL POUR DÉTERMINER LE POINT D'ÉCLAIR DE LIQUIDES INDIVIDUELS DANS UNE COUPE FERMÉE La portée et les définitions sont indiquées

Travail de laboratoire "Calcul du mode de transformation explosive du mélange air-carburant" Algorithme de calcul. La procédure de calcul est déterminée conformément à la méthodologie RD 03-40901 « Méthodologie d'évaluation de la

1. Fraction massique d'un élément dans une substance. La fraction massique d'un élément est sa teneur dans une substance en pourcentage en masse. Par exemple, une substance de composition C 2 H 4 contient 2 atomes de carbone et 4 atomes d'hydrogène. Si

UNIVERSITÉ AGRICOLE D'ÉTAT DE NOVOSIBIRSK FACULTÉ D'AGRONOMIE THÉORIE DE LA COMBUSTION ET DE L'EXPLOSION Collection de tâches et d'exercices pour effectuer des tests NOVOSIBIRSK 215 1 UDC 544.45 (75.) BBK 24.46,

UNIVERSITÉ TECHNIQUE OUVERTE D'ÉTAT DE RUSSIE DES COMMUNICATIONS DU MINISTÈRE DES COMMUNICATIONS DE LA FÉDÉRATION DE RUSSIE 18/16/2 Approuvé par le département "Génie thermique et hydraulique dans le transport ferroviaire"

MINISTÈRE DE L'ÉDUCATION ET DES SCIENCES DE RUSSIE Établissement d'enseignement budgétaire de l'État fédéral de l'enseignement professionnel supérieur "Université technique d'État d'Ukhta" (USTU)

Thème 4 Sécurité explosion et incendie en production Objectif : acquérir des compétences pratiques dans la réalisation de calculs techniques pour évaluer les risques d'explosion et d'incendie des entreprises industrielles. Planifier

La place de la discipline dans la structure du programme pédagogique La discipline "Théorie de la Combustion et de l'Explosion" est la discipline de la partie fondamentale. Le programme de travail est établi conformément aux exigences de l'État fédéral

CHIMIE GÉNÉRALE, INORGANIQUE ET PHYSIQUE TÂCHES INDIVIDUELLES Tâche 1 Calculer la quantité de chaleur nécessaire pour chauffer n mol de substance A d'une température de 298 K à une température T à une température constante

Cours 9 13. 4. 6 7.8. Calcul de la constante d'équilibre en fonction de la fonction de partage moléculaire Ζ. 7.9. Calcul d'équilibre de systèmes chimiques complexes. Tâche magistrale À R atm et T98 K pour la réaction gazeuse 1 SO + 5O

Ministère de l'éducation et des sciences de la Fédération de Russie Université d'État de l'Oural du Sud Département de la sécurité des personnes 6 (7) B16 M.Yu. Babkin, S.I. Borovik THÉORIE DE LA COMBUSTION ET DE L'EXPLOSION

Planification thématique en chimie pour l'année scolaire 2017-2018 Manuel de 9e année : O.S. GABRIELYAN. CHIMIE. 8ÈME ANNÉE. Moscou, DROFA, 2007-2012 Le contenu du matériel de formation Délais Minimum obligatoire INTRODUCTION.

MINISTÈRE DE L'ÉDUCATION ET DES SCIENCES DE LA FÉDÉRATION DE RUSSIE UNIVERSITÉ D'ÉTAT D'ARCHITECTURE ET DE CONSTRUCTION DE NOVOSIBIRSK (SIBSTRIN) Département de chimie CINÉTIQUE CHIMIQUE ET ÉQUILIBRE Tâches individuelles

TRAVAIL DE CONTRÔLE Thème : « Alcools monohydriques » 1 1. RAPPELER LES PROPRIÉTÉS CHIMIQUES ET L'OBTENTION DES ALCOOLS MONOTIQUES. 2. EFFECTUER LES TESTS SUGGÉRÉS 22 & 23 (Votre choix) PROPRIÉTÉS CHIMIQUES DES MONOALCOOLS

2.1. Masse des atomes et des molécules Pour mesurer les masses des atomes et des molécules en physique et en chimie, un système de mesure unifié a été adopté. Ces quantités sont mesurées en unités relatives d'unités de masse atomique. unité atomique

Test Éléments contrôlés de connaissance des tâches 1-2 Classification des substances organiques 3 Groupes fonctionnels des principales classes de composés organiques 4 Homologues et leurs noms 5 Isomères et leurs noms

MINISTRE DE L'ÉDUCATION ET DES SCIENCES DE LA FÉDÉRATION DE RUSSIE

Test final (1 semestre), option 1 1. Pour la réaction 2 HCl (g) \u003d H 2 + Cl 2, calculez :, K p, K s, KP, 625 si les données suivantes sont connues : H 289 U, H 625, A , HCl (g) Cl 2 (g) H 2 (g) H arr,

GÉORGIE. Tikhanovskaya L.M. Voropay BASES PHYSIQUES ET CHIMIQUES DU DÉVELOPPEMENT ET DE L'EXTINCTION DU FEU Vologda 2014 Ministère de l'éducation et des sciences de la Fédération de Russie Vologda State University G.A. Tikhanovskaïa

Planification thématique en chimie (étude externe) pour l'année académique 2016-2017 en 11e année Manuel : O.S. GABRIELYAN. CHIMIE. 11e année. UN NIVEAU DE BASE DE. Moscou, DROFA, 2007-2015 Contenu semestriel du matériel pédagogique

Agence fédérale pour l'éducation Établissement d'enseignement supérieur de l'État fédéral Université d'État de Novgorod nommée d'après Yaroslav le Sage Faculté

CALENDRIER ET PLANIFICATION THÉMATIQUE EN CHIMIE EN 10e PROMOTION ANNÉE ACADÉMIQUE 2009-2010. 2 heures par semaine. Programme pour écoles secondaires, gymnases, lycées. Classes de chimie 8-11, M. "Business Bustard", 2009. Manuel principal :

Agence fédérale pour l'éducation Établissement d'enseignement supérieur d'État Université d'État de Novgorod. Iaroslav le Sage Faculté des sciences naturelles

TRAVAIL DIAGNOSTIQUE en CHIMIE 10e année 6 avril 2011 Option 1 A1. Les substances organiques comprennent a) C 2 H 2 b) CaCO 3 c) C 2 H 5 OH d) CO e) C 2 H 5 NH 2 1) a, b, d 2) a, c, e 3) b, c , d 4) b,

TÂCHES pour la 2e étape de l'Olympiade "Premiers pas en médecine" en chimie Nom complet CLASSE ADRESSE DE L'ÉCOLE, TÉLÉPHONE Option 1 (60 points) PARTIE 1 (12 points) Lorsque vous remplissez les tâches de cette partie dans la feuille de réponses 1 sous le numéro

IA Gromchenko Collection de tâches en chimie pour la 8e année Centre d'éducation de Moscou 109 2009 1. Fraction de masse d'un élément. Calculs de formules. 1.1. Quelle substance a une molécule plus lourde : BaO, P 2 O 5, Fe 2 O 3 ? 1.2.

Examen final en chimie pour la 10e année de l'année académique 2017-2018 Option 1. Partie A. Lors de la réalisation des tâches de cette partie (A1-A15), parmi les quatre options proposées, sélectionnez-en une correcte. Sur le formulaire

MINISTÈRE DE L'ÉDUCATION ET DES SCIENCES DE LA FÉDÉRATION DE RUSSIE Établissement d'enseignement budgétaire de l'État fédéral de l'enseignement professionnel supérieur "UFA STATE AVIATION TECHNICAL

OPTION 1 1. Deux récipients d'une capacité de 0,2 et 0,1 litres sont séparés par un piston mobile qui ne conduit pas la chaleur. La température initiale du gaz dans les cuves est de 300 K, la pression est de 1,01 10 5 Pa. Le plus petit navire a été refroidi à 273 K, et le plus grand

1. La charge du noyau d'un atome de fer est : 1) +8 ; 2) +56 ; 3) +26 ; 4) +16. Version de démonstration du travail sur le test d'entrée en chimie Partie 1 2. Dans quelle rangée se trouvent les formules des substances à covalence uniquement

Billets d'examen en chimie 10e année Billet 1 1. Les principales dispositions de la théorie de la structure chimique des substances organiques A.M. Butlerov. Structure chimique comme ordre de connexion et influence mutuelle des atomes

Version de démonstration du travail en chimie pour le cours 0 classe Partie A .. Lorsque vous terminez la tâche dans la liste de réponses proposée, sélectionnez les deux bonnes et notez les numéros sous lesquels elles sont indiquées. Pour l'éthanol, ce qui suit est vrai


Saratov 2010

Compilé par: R.P. Volkov, chargé de cours à FGOU SPO "SGPPC nommé d'après Yu. A. Gagarin"

Réviseur interne : O.G. Stegalkina - conférencier

FGOU SPO "SPPK du nom de Yu.A. Gagarine"

Lignes directrices pour résoudre des problèmes et effectuer un travail indépendant sur le cours "Bases physiques et chimiques pour le développement et l'arrêt de la combustion dans un incendie" pour les étudiants de toutes les formes d'enseignement dans la spécialité "Sécurité incendie".

Les lignes directrices prennent en compte des exemples de résolution de problèmes typiques dans la section « Fondamentaux des processus de combustion. Bilan matière et thermique des processus de combustion" discipline "Bases physiques et chimiques du développement et de l'arrêt de la combustion dans un incendie" ; options données pour les tâches de résolution de problèmes indépendante.

Imprimé dans l'imprimerie du FGOU SPO "SGPPC nommé d'après Yu.A. Gagarine"

INTRODUCTION

Lignes directrices pour résoudre les problèmes et effectuer un travail indépendant sur la section «Principes fondamentaux des processus de combustion. Bilan matière et thermique des procédés de combustion" de la discipline "Bases physiques et chimiques du développement et de l'arrêt de la combustion en cas d'incendie" sont destinés à former des ingénieurs en sécurité incendie dans le cadre du programme de travail de la discipline "Bases physiques et chimiques du développement". et arrêt de la combustion en cas d'incendie » dans la spécialité 280104.

Les lignes directrices pour la résolution des problèmes sont élaborées en pleine conformité avec la norme éducative de l'État de l'enseignement professionnel supérieur, en tenant compte des spécificités activité professionnelle Employés GPS. Les tâches visent à consolider le cours théorique et les méthodes de calculs pratiques dans cette partie de la discipline. Des instructions méthodiques aideront les étudiants à maîtriser le matériel de la discipline étudiée, nécessaire au succès du travail d'un ingénieur en sécurité incendie dans n'importe quel domaine de son activité.

Les directives comprennent: de brèves dispositions théoriques, des dispositions générales pour le calcul du bilan matière et thermique des processus de combustion de substances gazeuses et condensées, la nature de la lueur de la flamme, la température de combustion, ainsi qu'un grand nombre d'exemples de résolution de problèmes typiques et référence informations nécessaires à la résolution des problèmes.

La structure et le contenu des lignes directrices pour la résolution de problèmes permettent aux stagiaires d'élaborer eux-mêmes le matériel de chaque section de la discipline.

En commençant à étudier le cours, il est nécessaire d'imaginer que la base de tous les phénomènes se produisant dans un incendie est le processus de combustion. La connaissance de l'essence de ce phénomène, des lois de la combustion, des mécanismes et des méthodes de sa terminaison est nécessaire au succès du travail d'un ingénieur en sécurité incendie dans n'importe quel domaine de son activité.

1. Écrire des formules structurales, établir des équations pour les réactions de combustion de substances combustibles dans l'air et calculer des coefficients stoechiométriques.

1.1. amylbenzène, acide abiétique, allylamine;

1.2. amyl diphényl, acide adipique, allyl isothiocyanate;

1.3. amylène, acide acrylique, alnapht;

1.4. amylnaphtalène, acétate d'allyle, altax;

1.5. amyltoluène, diacétate d'allylidène, amylamine;

1.6. anthracène, caproate d'allyle, nitrate d'amyle;

1.7. acénaphtène, alcool allylique, nitrite d'amyle;

1.8. acétylène, acétate d'amyle, sulfure d'amyle;

1.9. benzène, butyrate d'amyle, trichlorosilane d'amyle;

1.10. le butylbenzène, l'amylxylyl éther, l'amylchloronaphtalène;

1.11. butylcyclohexane, amillaurate, aminalon;

1.12. butylcyclopetane, amyl méthyl cétone, colorant amino azoïque;

1.13. l'hexadécane, l'amyloléate, l'acide aminocaproïque ;

1.14. hexane, salicylate d'amyle, acide aminopélargonique;

1.15. hexylcyclopentane, stéarate d'amyle, aminocyclohexane;

1.16. heptadécane, amylphénylméthyléther, ampicilline;

1.17. heptane, éther amniphénylique, anginine;

1.18. décane, formiate d'amyle, aniline;

1.19. diamylbenzène, anisole, antitrimid;

1.20. diamylnaphtalène, acétal, atophane;

1.21. divinylacétylène, acétaldéhyde, acéclidine;

1.22. dihydrocyclopentadiène, acétylacétone, acétanilide;

1.23. diisobutylène, acide acétysalicylique, chlorure d'acétyle;

1.24. diisopropylbenzène, acétyltributylcitrate, acétoacétanilide;

1.25. le diméthylènecyclobutane, l'acétométhoxane, l'acétonitrile ;

1.26. ditolylméthane, acétone, acétoxime;

1.27. diphényle, acétonylacétone, acétoéthylamide;

1.28. diphénylméthane, alcool acétopropylique, benzamide;

1.29. diéthylcyclohexane, éther acétoacétique, benzyldiéthylamine;

1.30. dodécane, acétophénone, benzylthiol;

1.31. isobutylbenzène, benzaldéhyde, chlorure de benzyle;

1.32. isobutylcyclohexane, benzantrone, cyanure de benzyle;

1.33. isooctane, benzhydrol, benzimidazole;

1.34. isopentane, acétate de benzyle, benzoate de sodium;

1.35. isoprène, benzoate de benzyle, chlorure de benzoyle;

1.36. isopropénylbenzène, salicylate de benzyle, benzoxazolone;

1.37. isopropylacétylène, benzyl cellosolve, benzène sulfazide;

1.38. le méthylcyclohexane, l'éther benzyléthylique, le benzènesulfamide;

1.39. méthylcyclopentane, acide benzylsuccinique, acide benzènesulfonique;

1.40. octyltoluène, acétate de méthoxybutyle, benzonitrile.

2. Écrire des formules structurelles et déterminer lors de la combustion de quelle substance combustible le plus grand nombre de moles de produits de combustion sera libéré ?

2.1. la benzophénone et l'acide benzophénone tétracarboxylique;

2.2. bornéol et butanal;

2.3. l'acide butanoïque et l'acétate de butyle ;

2.4. butylacétylricinoléate et butylacétoacétate

2.5. le sébacate de butylbenzyle et le benzoate de butyle ;

2.6. le butyrate de butyle et l'éther butylvinylique;

2.7. le butyl glycol et l'acétate de butyl glycol ;

2.8. l'éther de butylglycide et l'adipate de butyldiéthyle;

2.9. le butylisovalérate et le butylcapronate;

2.10. le butyl carbitol et le lactate de butyle ;

2.11. le laurate de butyle et le méthacrylate de butyle ;

2.12. la butylméthylcétone et l'oléate de butyle;

2.13. le propionate de butyle et l'oléate de butylricino;

2.14. le stéarate de butyle et l'éther de butylphényle ;

2.15. le formiate de butyle et l'acétaldéhyde de butyléthyle;

2.16. la butyléthylcétone et l'éther butyléthylique;

2.17. acide valérique et aldéhyde valérique;

2.18. vanilline et acétate de vétiveryle;

2.19. l'alcool de vétiver et l'acétate de vétinyle ;

2.20. vétinone et éther vinylallylique;

2.21. l'acétate de vinyle et le butyrate de vinyle ;

2.22. l'éther vinylisobutylique et l'éther vinylisooctylique;

2.23. l'éther isopropylique de vinyle et le crotonate de vinyle ;

2.24. la vinyl méthyl cétone et le vinyl oxyéthyl méthacrylate;

2.25. l'éther octadécylique vinylique et le propionate de vinyle;

2.26. l'éther vinyltriméthylnonylique et l'éther vinyléthylique;

2.27. éther vinyléthylique et acide tartrique;

2.28. vitamine A (acétate) et vitamine C;

2.29. acide gallique et hexanal;

2.30. l'acide hexanoïque et l'acétate d'hexyle;

2.31. butyrate d'hexyle et hexahydrophtalate de diéthyle hexyle;

2.32. le méthacrylate d'hexyle et l'hexylméthylcétone;

2.33. alcool hexylique et propionate d'hexyle;

2.34. formiate d'hexyle et cellosolve d'hexyle;

2.35. l'héliotropine et l'alcool heptadécylique;

2.36. heptanal et heptylacétate;

2.37. le butyrate d'heptyle et l'heptyl diphényl cétone;

2.38. l'heptyl isobutyl cétone et l'heptyl méthyl cétone;

2.39. alcool heptylique et propionate d'heptyle;

2.40. formiate d'heptyle et hydroquinone.


SOLUTION.

1. Nous composons les équations des réactions de combustion des gaz combustibles du mélange dans l'air :

C 2 H 2 + 2,5 (O 2 + 3,76 N 2) \u003d 2 CO 2 + H 2 O + 2,5*3.76N2 ,

C 3 H 8 + 5 (O 2 + 3,76 N 2) \u003d 3 CO 2 + 4 H 2 O + 5*3.76N2.

2. Calculer les volumes théoriques d'air et de produits de combustion lors de la combustion complète de 1 m 3 de mélange gazeux (formules 8 et 15) :

3. Calculez les volumes réels d'air et de produits de combustion en tenant compte d'un excès d'air de 40 % (α = 1,4).

4. Étant donné que le volume du mélange combustible était de 10 m 3 , les volumes réels d'air et de produits de combustion seront respectivement de 176,7 et 192,9 m 3 .

RÉPONSE: La combustion de 10 m 3 d'un mélange gazeux complexe nécessite 176,7 m 3 d'air, tandis que 192,9 m 3 de produits de combustion se forment.

EXEMPLE:Déterminer les volumes d'air et de produits de combustion lors de la combustion de 2 kg d'une substance combustible ayant une composition élémentaire : C = 50 % ; H = 10 % ; N = 10 % ; cendres = 12 % ; humidité = 18%. Supposez que l'air et les produits de combustion sont dans des conditions normales.

SOLUTION:

1. Pour résoudre le problème, nous utilisons les formules (9) et (16).


Lors de la combustion de 2 kg de substance combustible, 14,34 et 16,14 m 3 d'air et de produits de combustion se forment respectivement.

RÉPONSE: Lors de la combustion de 2 kg de substance combustible, 14,34 m 3 d'air sont consommés et 16,14 m 3 de produits de combustion se forment.

TÂCHES POUR UNE SOLUTION INDÉPENDANTE

1. Déterminer le volume d'air nécessaire à la combustion de 50 m 3 d'acétylène à α=1, 7.

2. Déterminer les volumes d'air, de produits de combustion et le pourcentage de produits de combustion dans 2 m 3 d'éthane. Prendre la température des produits de combustion à 1200 K, la pression à 101,3 kPa, l'excès d'air α=1,2.

3. Déterminer le volume d'air nécessaire à la combustion de 15 m 3 de butane à une température de 10С et une pression de 750 mm Hg. Art., si la combustion se déroule avec un coefficient d'excès d'air égal à 1,4 (α=1,4).

6. Calculer la quantité d'amylbenzène qui peut brûler dans une pièce fermée d'un volume de 200 m 3 si la combustion s'arrête à une teneur résiduelle en oxygène de 12 %. La température initiale dans la pièce est de 24 ° C, la pression est de 98 kPa.

7. Déterminer combien d'acétate de butyle peut brûler dans une pièce d'un volume de 200 m 3 si sa combustion s'arrête à une teneur en oxygène de l'air égale à 13,8 % (conditions normales).

8. Déterminer les volumes de produits de combustion et d'air lors de la combustion de 7 kg d'hexane. Le processus de combustion s'est déroulé à une température de 33°C et une pression de 730 mm. rt. Art. La température des produits de combustion est supposée être de 1300 K.

9. Déterminer les volumes de produits de combustion et d'air lors de la combustion de 11 kg d'acétone. Le processus de combustion s'est déroulé à une température d'environ 30 C et une pression de 720 mm Hg. Art. La température des produits de combustion est supposée être de 1300 K.

10. Déterminer le volume de produits de combustion et d'air lors de la combustion de 17 kg de toluène. Le processus de combustion s'est déroulé à une température d'environ 30 C et une pression de 745 mm Hg. Art. La température des produits de combustion est supposée être de 1100 K.

11. Calculer le volume d'air et le volume des produits de combustion lors de la combustion complète de 6 kg de cellulose, composée de 80 % de carbone, 13 % d'hydrogène et 7 % d'oxygène, si la combustion se produit à une température de 25°C et une pression de 95 kPa . Le coefficient d'excès d'air est de 1,4.

12. Déterminer le volume d'air nécessaire à la combustion de 6 kg d'éther diéthylique à une température de 15°C et une pression de 750 mm Hg. Art. Le coefficient d'excès d'air était de 1,3.

13. Déterminez la quantité de benzène brûlée dans une pièce fermée d'un volume de 180 m 3, si l'on sait que sa combustion s'est arrêtée lorsque la teneur en oxygène de l'air était de 14,6%. La température avant l'incendie était de 19 o C et la pression de 100 kPa.

15. Déterminer le coefficient d'excès d'air si 212 m 3 d'air sont consommés pour la combustion de 8 kg d'acétate d'éthyle à une température de 25°C et une pression de 760 mm Hg. Art.

16. Calculer le coefficient d'excès d'air et le pourcentage de dioxyde de carbone dans les produits de combustion si 70 m 3 d'air sont consommés pour la combustion complète de 4 kg d'éther éthyl propylique (C 5 H 12 O) à une température de 22 ° C et une pression de 92 kPa.

17. 3 kg d'acroléine brûle à une température de 21°C et une pression de 98 kPa. Calculez le volume d'air qui est passé dans les produits de combustion et le pourcentage d'eau qu'ils contiennent si la combustion se produit avec un excès d'air (le coefficient d'excès d'air est de 1, 2).

20. Calculer le volume d'un mélange gazeux composé de 45 % de butane, 30 % de méthane, 20 % d'acétylène et 5 % d'oxygène, si 80 m 3 d'air sont consommés pour sa combustion dans des conditions normales. Le coefficient d'excès d'air est de 1,6.

22. Calculer le volume d'air et le volume des produits de combustion lors de la combustion complète de 7 m 3 d'un mélange gazeux composé de 57 % d'hydrogène, 18 % de monoxyde de carbone et 25 % de méthane, si la combustion se déroule avec un excès d'air (le coefficient d'excès d'air est de 1,3 ).

23. Calculer le volume d'air et le volume des produits de combustion lors de la combustion complète de 6 kg de cérésine, constituée de 80 % de carbone, 15 % d'hydrogène et 5 % d'oxygène, si la combustion se produit à une température de 25°C et une pression de 95 kPa. Le coefficient d'excès d'air est de 1,5.

25. Déterminer le volume et la composition des produits de combustion (en % vol.) du mélange gazeux (tableau 4), si la combustion se produit à un coefficient d'excès d'air α (voir tableau 4).

Tableau 4

Composition du mélange, % Numéro de travail
25.1 25.2 25.3 25.4 25.5 25.6 25.7 25.8 25.9 25.10
monoxyde de carbone - - - - - -
Hydrogène - - - - - - -
Méthane - - - - - -
Éthane - - - - - --
Propane - - - - - - - - -
Butane - - - - - - - -
Éthylène - - - - -
propène - - - - - - - -
Acétylène - - - - - - -
Gaz carbonique - -
Azote - - - -
Oxygène - - -
α 1,2 1,3 1,1 1,2 1.2 1,2 1,4 1,1 1,3

Tableau 5

numéro de travail Substance Composition élémentaire de la matière à C Masse de substance, kg
C H O S O cendre
26.1 Cérésine
26.2 Charbon
26.3 Bois
26.4 Essence
26.5 Huile
26.6 essence
26.7 Kérosène
26. 8 schiste bitumineux
26.9 Charbon
26.10 Anthracite 0,2 5,8

27. Déterminer la nature de la lueur de la flamme de l'éthylbenzène.

28. Déterminer la nature de la lueur de la flamme de l'acide acétique.

29. Déterminer la nature de la lueur de la flamme d'hexane.

30. Déterminer la nature de la lueur de la flamme de l'alcool amylique.

31. Déterminez la nature de la lueur de la flamme de butane.

32 . Déterminer la nature de la lueur de la flamme de benzène.

CALCUL DE LA TEMPÉRATURE DE COMBUSTION

La température de combustion s'entend comme la température maximale à laquelle les produits de combustion sont chauffés. En ingénierie et lutte contre l'incendie, une distinction est faite entre les températures de combustion théoriques, calorimétriques, adiabatiques et réelles.

Température de combustion théorique est la température à laquelle la chaleur de combustion dégagée d'un mélange de composition stoechiométrique est dépensée pour le chauffage et la dissociation des produits de combustion. En pratique, la dissociation des produits de combustion commence à des températures supérieures à 2 000 K.

R

Température de combustion calorimétrique est la température atteinte lors de la combustion d'un mélange combustible stoechiométrique avec une température initiale de 273 K et en l'absence de pertes dans l'environnement.

Température de combustion adiabatique est la température de combustion complète des mélanges de toute composition en l'absence de pertes de chaleur dans l'environnement.

Température de combustion réelle est la température de combustion atteinte dans un feu réel. Elle est bien inférieure à la théorique, calorimétrique et adiabatique, car dans des conditions réelles, jusqu'à 40% de la chaleur de combustion est généralement perdue par rayonnement, sous-combustion, chauffage de l'air en excès, etc.

La détermination expérimentale de la température de combustion pour la plupart des substances combustibles présente des difficultés importantes, en particulier pour les liquides et les matériaux solides. Cependant, dans un certain nombre de cas, la théorie permet de calculer la température de combustion des substances avec une précision suffisante pour la pratique, en se basant uniquement sur la connaissance de leur formule chimique, de la composition du mélange combustible initial et des produits de combustion.

Dans le cas général, la dépendance suivante est utilisée pour les calculs (approximatif, puisque C p \u003d f (T)):

Q pg \u003d V pg * C p * T g,

où Q pg - enthalpie des produits de combustion;

V pg - la quantité de produits de combustion, m 3 /kg ;

C p - la capacité calorifique volumétrique moyenne du mélange de produits de combustion dans la plage de températures allant de T 0 à T g, kJ / (m 3 * K);

T g - température de combustion, K.

L'enthalpie des produits de combustion est déterminée à partir de équations du bilan thermique:

Q pg \u003d Q H + Q ref - Q sueur, (24)

Q sueur = Q u + Q nedo + Q dis Avec , (25)

Q utiliser est la chaleur d'évaporation;

Q sueur- perte de chaleur due à radiation, sous-combustion et dissociation produits de combustion.

Selon le type de déperditions thermiques prises en compte dans la zone de combustion (par rayonnement, sous-combustion, dissociation des produits de combustion), l'une ou l'autre température est calculée.

Lors de la combustion cinétique de mélanges gaz-vapeur-air, les pertes de chaleur de la zone de combustion sont négligeables, par conséquent, pour ces mélanges, la température de combustion réelle est proche de l'adiabatique, qui est utilisée dans les calculs d'ingénierie incendie.

Il est très difficile de déterminer la capacité calorifique moyenne d'un mélange de produits de combustion. En gros, l'enthalpie d'un mélange de produits de combustion peut être exprimée comme la somme des enthalpies de ses composants :

Qpg =Σ (V pg) je (С р) je*T g, (26)

å ×
rien

(V pg) je est la quantité du i-ème composant des produits de combustion ;

C p est la capacité calorifique volumique moyenne du ième composant à T g et pression constante

T g- température de combustion.

Lors du calcul de la température de combustion, la valeur est utilisée Qn(pouvoir calorifique inférieur), car à la température de combustion l'eau est à l'état gazeux.

Les valeurs du pouvoir calorifique inférieur d'une substance (effet thermique d'une réaction chimique) sont données dans la littérature de référence, et peuvent également être calculées à partir du corollaire de la loi de Hess :

Q n \u003d (Σ ΔН i *n i) prod - (ΣΔН i * n i) ref, où (27)

ΔН i est la chaleur de formation de la ième substance,

n je est le nombre de moles de la ième substance.

Selon corollaire de la loi de Hess l'effet thermique d'une réaction chimique est égal à la différence entre les sommes des chaleurs de formation des produits de réaction et des chaleurs de formation des matières premières. Rappelez-vous du cours de chimie que la chaleur de formation des substances simples (oxygène, azote, etc.) est nulle.

Par exemple, calculons la chaleur de combustion (effet thermique) de l'éthane :

C2H6 + 3,5*(O 2 + 3,76N 2) \u003d 2 CO 2 + 3 H 2 O + 3,76 *3.5N2.

Le pouvoir calorifique inférieur, selon la conséquence de Hess, est égal à :

Q n \u003d ΔH CO 2 * n CO 2 + Δ H H 2 O * n H 2 O - ΔH C 2 H 6 * n C 2 H 6 (28)

En remplaçant les valeurs de la chaleur de formation de CO 2 , H 2 O, C 2 H 6 à partir des données de référence, déterminez le pouvoir calorifique inférieur de l'éthane.

Lorsqu'un mélange de substances individuelles est brûlé, la chaleur de combustion de chaque composant est d'abord déterminée, puis additionnée en tenant compte du pourcentage de chaque substance combustible dans le mélange:

Si le carburant est une substance complexe et que sa composition élémentaire est donnée en pourcentage de masse, la formule de Mendeleïev est utilisée pour calculer la chaleur de combustion :

Q n c m \u003d 339,4 * C + 1257 * H-108,9 (O-N-S) -25 (9 * H + W), kJ / kg (30)

où C, H, O, N, S est le pourcentage de cet élément dans la substance combustible ;

O– taux d'humidité en masse. %.

Pour calculer la température de combustion, nous composons l'équation du bilan thermique, en supposant que la chaleur dégagée à la suite de la combustion chauffe les produits de combustion à partir de la température initiale T 0 jusqu'à la température T M.:

Q n (1-η) \u003d Σs rpg je * V pg je (T g -T 0)

Où η coefficient de perte de chaleur (part de la perte de chaleur due au rayonnement, ainsi qu'à la suite d'une combustion incomplète);

avec RPG je capacité calorifique du i-ème produit de combustion à pression constante, kJ/molK ;

V pg i - le volume du i-ème produit de combustion, m 3.

Calcul du volume des produits de combustion ( CO2, H2O, SO2, N2) s'effectue selon les formules suivantes :

À partir de l'équation du bilan thermique :

La difficulté de déterminer la température de combustion à l'aide de cette formule est que la capacité calorifique d'un gaz dépend de la température. Comme les gaz sont chauffés par la température T 0 jusqu'à la température T g, alors dans la formule (36) il faut substituer la valeur moyenne de la capacité calorifique dans cette plage de température. Mais la température de combustion nous est inconnue et nous voulons la trouver. Dans ce cas, vous pouvez procéder comme suit. La valeur moyenne de la température de combustion de la plupart des substances dans l'air est d'environ 1500 K. Par conséquent, avec une petite erreur dans la détermination T g pour les calculs, nous pouvons prendre la valeur moyenne de la capacité calorifique dans la plage de températures 273–1500 K. Ces valeurs pour les principaux produits de combustion sont données dans le tableau. 6.

Tableau 6

Capacités calorifiques moyennes des principaux produits de combustion dans la plage de température 273–1500 o C





La valeur moyenne de la capacité calorifique de certaines substances gazeuses dans différentes plages de température est également indiquée dans le tableau. III demandes.

Considérons des exemples de résolution de problèmes pour calculer la température de combustion.

TÂCHES POUR UNE SOLUTION INDÉPENDANTE

1. Dans ce cas, dans des conditions d'incendie, lors de la combustion du butane, plus de chaleur sera dégagée : avec combustion complète ou incomplète, procédant par la réaction C4H10 + 4,5O2 4CO + 5H 2 O. La réponse doit être confirmée par un calcul utilisant la loi de Hess.

2. Calculez la chaleur de formation de l'acétylène à partir des éléments si sa chaleur de combustion est de 1411,2 kJ/mol.

3. Déterminez la chaleur de combustion de 12 kg de benzène, si la chaleur de sa formation est de 82,9 kJ/mol, la chaleur de formation du dioxyde de carbone est de 396,9 kJ/mol, la chaleur de formation de la vapeur d'eau est de 242,2 kJ/mol .

4. Déterminez la chaleur de formation de l'acide pimélique (C 7 H 12 O 4), si la chaleur de combustion est de 3453,5 kJ / mol.

5. Déterminez la chaleur de combustion de l'acide salicylique si la chaleur de sa formation est de 589,5 kJ/mol.

6. Calculez la chaleur de formation du méthane si la combustion de 10 g de celui-ci dans des conditions standard dégage 556,462 kJ de chaleur.

7. Déterminez la chaleur de combustion de l'alcool benzylique (C 7 H 8 O), si la chaleur de sa formation est de 875,4 kJ / mol.

8. Lors de la formation d'octane, 208,45 kJ / mol de chaleur sont dégagés des éléments. Calculer sa chaleur de combustion.

9. La chaleur de formation de l'acétone est de -248,28 kJ/mol. Déterminez sa chaleur de combustion et la quantité de chaleur dégagée lors de la combustion de 30 g de la substance.

11. Déterminer le pouvoir calorifique de la sulfophénylhydrazine (C 6 H 8 O 3 N 2 S) en tenant compte des pertes dues à l'évaporation de l'eau. La teneur en humidité de la substance est de 20%.

12. Déterminer le pouvoir calorifique de la 4, 4 / -diaminodiphénylsulfone (C 12 H 12 O 2 N 2 S) sans tenir compte des pertes par évaporation d'humidité.

13. Déterminez la chaleur de combustion du 4, 6-diméthylhexahydro-1, 3, 5-triazinethion-2 (C 5 H 9 N 3 S) selon les formules de D. I. Mendeleev.

14. Déterminez la chaleur de combustion de l'acide diaminomésitylène-6-sulfonique (C 9 H 14 O 3 N 2 S), si la teneur en humidité de la substance est de 35%.

15. Déterminez le pouvoir calorifique le plus bas de la composition du bois : C - 41,5 % ; H-6 % ; 0-43 % ; N, 2%; W– 7,5 %.

16. Déterminer la température de combustion théorique de l'acétone en utilisant les capacités calorifiques moyennes.

17. Déterminer la température de combustion théorique du pentane en utilisant les capacités calorifiques moyennes.

18. Déterminer la température de combustion théorique de l'octane en utilisant les capacités calorifiques moyennes.

19. Déterminer la température de combustion théorique du benzène en utilisant les capacités calorifiques moyennes.

20. En utilisant la méthode des approximations successives, calculez la température adiabatique de la combustion du propanol.

21. Calculez la température de combustion d'un mélange stoechiométrique de substance combustible avec de l'air (tableau 7).

Tableau 7

22. À l'aide de la méthode des approximations successives, calculez la température de combustion réelle d'une substance combustible (tableau 8), si la combustion se produit à un coefficient d'excès d'air α et que la proportion de perte de chaleur par rayonnement est η.

Tableau 8

numéro de travail Nom de la substance Composition élémentaire de la matière, poids. % α η
C H O S N O cendre
22.1 Anthracite 0,5 1,0 21,5 1,1 0,2
22.2 Schiste bitumineux 24,2 1,8 4,5 3,0 2,0 39,5 1,2 0,3
22.3 Kérosène 13,7 0,3 - - - 1,3 0,4
22.4 Essence 8,0 5,0 - 2,0 1,4 0,3
22.5 Huile de sel 86,0 12,0 1,2 0,8 - - - 1,5 0,2
22.6 essence - - 1,6 0,3
22.7 Bois - - 1,7 0,4
22.8 Charbon - 1,8 0,3
22.9 Cérésine - - - - 1,7 0,2
22.10 Schiste bitumineux 1,6 0,3

23. Déterminez la température de combustion théorique du composé de caoutchouc : AVEC = 80 %, H= 15 %, S = 2 %, O = 1 %, N = 2 %.

24 . Déterminez la température de combustion réelle du papier de composition: C \u003d 55%, H \u003d 25%, N \u003d 3%, O \u003d 15%, H 2 O \u003d 2%, si la perte de chaleur due à la sous-combustion était η X=0,15, dû au rayonnement η izl=0,20.

25. Déterminez la température de combustion réelle de la composition plastique : C = 70 %, H = 20 %, N = 5 %, O = 2 %, les composants non combustibles (charges) s'élevaient à 3 % /, si la perte de chaleur due à la sous-combustion s'élevait à η X=0,20, dû au rayonnement η izl=0,25. Coefficient d'excès d'air α = 1, 4.

APPENDICE

Liste des désignations acceptées

n est le nombre de moles de la substance ;

β est le coefficient stoechiométrique ;

V à théor- théoriquement nécessaire à la combustion, m 3 ;

V en ré- le volume réel (pratique) d'air qui est allé à la combustion, m 3;

V pg t- volume théorique des produits de combustion, m 3 ;

R– pression de gaz, Pa ;

P 0– pression initiale (atmosphérique), Pa;

J est la température de la substance, K;

Q est la quantité de chaleur, J;

Vi- le volume de la ième substance gazeuse, m 3, kmol;

α - coefficient d'excès d'air;

m est la masse de la substance, kg;

M est la masse d'un kmole d'une substance, kg/kmol;

Qn– pouvoir calorifique inférieur de la substance, kJ/mol, kJ/kg ;

Salut- enthalpie de la ième substance, kJ / mol, kJ / m 3;

T g– température de combustion, K ;

IPC est la capacité calorifique du i-ième gaz à pression constante, kJ/mol*K ; kJ/m3 ;

η est le coefficient de perte de chaleur.

Tableau I

Constantes physiques de base de certains gaz

Afin de simplifier le calcul, toutes les substances combustibles sont divisées en trois types: individuels, complexes, mélanges de gaz combustibles (tableau 1.2.1).

Tableau 1.2.1

type de carburant

Formules de calcul

Dimension

substance individuelle

(1.2.2)

Substance de composition complexe

(1.2.3)

(1.2.4)

(1.2.5)

mélange de gaz

(1.2.7)


(1.2.8)

Ici
- volume théorique des produits de combustion ;
- quantité du ième produit de combustion dans l'équation de réaction, kmol ; - quantité de carburant, kmol; - volume de 1 kmole de gaz ;
est le poids moléculaire du carburant ;
-volume du ième produit de réaction ; C, H, S, O, N - la teneur en éléments correspondants (carbone, hydrogène, soufre, oxygène et azote) dans la substance combustible, % en poids ; -teneur du i-ème composant combustible dans le mélange gazeux, % vol. ;
- contenu jee composant non combustible entrant dans la composition du mélange gazeux, % vol.

Le volume pratique (total) des produits de combustion se compose du volume théorique des produits de combustion et de l'excès d'air

(1.2.9)


(1.2.10)

Composition des produits de combustion, c'est-à-dire le contenu du i-ème composant est déterminé par la formule

(1.2.11)


- contenu je-ème composant dans les produits de combustion, % vol. ;

- le volume je- ème composant, m 3 , kmol;

- le volume total des produits de combustion, m 3, kmol.

Lors de la combustion dans un excès d'air, les produits de combustion contiennent de l'oxygène et de l'azote.

(1.2.12)

(1.2.13)

- volume théorique d'azote dans les produits de combustion, m 3, kmol.

(1.2.14)

Exemples

Exemple 1. Quelle quantité de produits de combustion sera libérée lors de la combustion de 1 m 3 d'acétylène dans l'air si la température de combustion était de 1450 K.

Le carburant est un composé chimique individuel (formule 1.2.1). Nous écrivons l'équation de la réaction chimique de combustion

C2H2+
O2+
N 2 \u003d 2CO 2 + H 2 O +
N 2

Le volume des produits de combustion dans des conditions normales

m3 / m3

Le volume des produits de combustion à 1450 K


m3 / m3

Exemple 2. Déterminer le volume des produits de combustion lors de la combustion de 1 kg de phénol, si la température de combustion est de 1200 K, la pression est de 95000 Pa, le coefficient d'excès d'air est de 1,5.

Le carburant est un composé chimique individuel (formule 1.2.2). Nous écrivons l'équation de la réaction chimique de combustion

C6H5OH+
O2+
N 2 \u003d 6CO 2 + 3H 2 O +
N 2

Le poids moléculaire du carburant est de 98.

m3/kg

Volume d'air pratique dans des conditions normales

Le volume des produits de combustion dans des conditions données

m3 / m3

Exemple 3. Déterminer le volume de produits de combustion lors de la combustion de 1 kg de masse organique de la composition : C-55%, O-13%, H-5%, S-7%, N-3%, W 17% , si la température de combustion est de 1170 K, coefficient d'excès d'air - 1,3.

Matière combustible de composition complexe (formules 1.2.3 - 1.2.6). Composition théorique des produits de combustion dans des conditions normales

m3/kg

m3/kg

Volume théorique total des produits de combustion dans des conditions normales

\u003d 1 + 0,8 + 0,05 + 4,7 \u003d 6,55 m 3 / kg

Volume pratique de produits de combustion dans des conditions normales

=6,55+0,269
(1,3-1) \u003d 6,55 + 1,8 \u003d 8,35 m 3 / kg

Volume pratique de produits de combustion à température de combustion

=
m3/kg.

Exemple 4. Calculer le volume des produits de combustion lors de la combustion de 1 m 3 d'un mélange gazeux constitué de C 3 H 6 -70%, C 3 H 8 -10%, CO 2 -5%, O 2 -15%, si la température de combustion est de 1300 K, coefficient d'excès d'air - 2,8. Température ambiante 293 K.

Le carburant est un mélange de gaz (formule 1.2.7).

Le volume des produits de combustion est déterminé par la formule (1.2.8)

m3 / m3

m3 / m3

Étant donné que le mélange gazeux contient de l'oxygène, il oxydera une partie des composants combustibles, par conséquent, la consommation d'air diminuera (formule 1.1.5).

Dans ce cas, il est plus pratique de déterminer le volume théorique d'azote par la formule (1.2.14)

m3 / m3

Volume théorique des produits de combustion

Volume pratique des produits de combustion

Le volume des produits de combustion à une température de 1300 K

m3 / m3.

Exemple 5. Déterminer la composition des produits de combustion de la méthyléthylcétone.

Avec une telle formulation du problème, il est rationnel de déterminer directement à partir de l'équation de combustion le volume de produits en kmol dégagé lors de la combustion de 1 kmol de carburant

kmolya ;
kmolya ;
kmolya ;
kmolya.

D'après la formule (1.2.11) on trouve la composition des produits de combustion

Exemple 6. Déterminer le volume et la composition des produits de combustion de 1 kg d'huile minérale de composition : C-85%, H-15%, si la température de combustion est de 1450 K, le coefficient d'excès d'air est de 1,9.

Solution. À l'aide des formules (1.2.3 - 1.2.6), nous déterminons le volume des produits de combustion

m3/kg

m3/kg

m3/kg

Volume théorique des produits de combustion dans des conditions normales

Volume pratique des produits de combustion dans des conditions normales formule (1.2.10)

Le volume des produits de combustion à une température de 1450 K

m3/kg

Évidemment, la composition des produits de combustion ne dépend pas de la température de combustion, il est donc conseillé de la déterminer dans des conditions normales. Par les formules (1.2.11;1.2.13)

;
;


Exemple 7. Déterminer la quantité d'acétone brûlée, kg, si le volume de dioxyde de carbone libéré, ramené aux conditions normales, était de 50 m 3.

Nous écrivons l'équation de la réaction de combustion de l'acétone dans l'air

Il résulte de l'équation que lors de la combustion à partir de 58 kg (poids moléculaire de l'acétone)
m 3 de dioxyde de carbone. Ensuite, pour la formation de 50 m 3 de dioxyde de carbone, le Mg du combustible doit réagir

kg

Exemple 8. Déterminer la quantité de composition de matière organique brûlée C-58%, O-22%, H-8%, N-2%, W-10% dans une pièce d'un volume de 350 m 3 si la teneur en dioxyde de carbone était de 5 %.

Solution. Déterminer la quantité de dioxyde de carbone libéré

m 3.

Selon la formule (1.2.6), pour une substance de composition complexe, on détermine le volume de CO 2 dégagé lors de la combustion de 1 kg de carburant,

m3/kg.

Déterminer la quantité de matière brûlée

kg.

Exemple 9. Déterminez le moment où la teneur en dioxyde de carbone dans une pièce d'un volume de 480 m 3 à la suite de la combustion de bois (C-45%, H-50%, O-42%, W-8%) a été 8% si le taux de combustion spécifique à la masse bois 0,008 kg / (m 2 s) et la surface de combustion est de 38 m 2. Lors de la résolution des échanges gazeux avec l'environnement, ne tenez pas compte de la dilution résultant de la libération de produits de combustion.

La dilution des produits de combustion n'étant pas prise en compte, on détermine le volume de dioxyde de carbone dégagé suite à la combustion, correspondant à 8% de sa teneur dans l'atmosphère

m 3

À partir de l'expression (1.2.3), nous déterminons la quantité de matière combustible qui doit brûler pour libérer un volume donné de dioxyde de carbone

kg.

Le temps de combustion est déterminé en fonction du rapport


,

- temps de combustion ;

mg- masse de bois brûlé, kg;

- taux de combustion massique du bois, kg / (m 2 s);

F- surface de combustion, m 2 ;

min.

Mission pour un travail indépendant

Tâche 3 : Déterminer le volume des produits de combustion lors de la combustion de 1 kg d'une substance donnée, si la température de combustion ... K, pression ... mm Hg,  = ... .

Substance

J p.g., K

R, mmHg.

amylbenzène

Alcool N-amylique

Acétate de butyle

Alcool butylique

l'éther diéthylique

Èsprit blanc

éthylène glycol

alcool tert-amylique

Alcool méthylique

Amylméthylcétone

Butylbenzène

Éther butylvinylique

Éthanol

Alcool butylique

Tâche 4: Déterminer le volume et la composition (% vol.) des produits de combustion libérés lors de la combustion de 1 m 3 de gaz combustible, si la température de combustion était ... K, la pression ... mm Hg.

Substance

J p.g., K

R, mmHg.

Acétylène

monoxyde de carbone

sulfure d'hydrogène

Acétylène

monoxyde de carbone

sulfure d'hydrogène

monoxyde de carbone

Acétylène

Acétylène

monoxyde de carbone

Nouveau sur place

>

Le plus populaire